Liver Flashcards

You may prefer our related Brainscape-certified flashcards:
1
Q
  1. A 48-year-old presents with a right upper quadrant mass. He has a history of episodes of constipation with no weight loss. CT shows a 6cm mass in the left lobe of the liver, with peripheral globular cloud-like enhancement. Delayed phase images show the mass ‘has filled in’ with contrast, with minimal low attenuation central remaining. T2 MRI shows a well-defined high signal mass in the left lobe of the liver. Which is the most likely diagnosis?

A. Hypervascular liver metastasis

B. Focal Nodular Hyperplasia (FNH)

C. Hepatocellular Carcinoma (HCC)

D. Cavernous haemangioma

E. Metastatic colon cancer

A

D. Cavernous haemangioma

Peripheral nodular enhancement with central fill-in is characteristic.

How well did you know this?
1
Not at all
2
3
4
5
Perfectly
2
Q
  1. A 40-year-old woman presents with some poorly localizing abdominal pain and is investigated with CT. A non-contrast study shows a 4cm isoattenuated mass in the left lobe. Arterial phase images show the mass to enhance with a small lower attenuation focus centrally. On the PV phase, the mass is isoattenuating but the central low attenuated focus persists. What is the diagnosis?

A. Regional nodule

B. FNH

C. HCC

D. Hypervascular metastasis

E. Cavernous haemangioma

A

B. FNH

Typically, iso on non-contrast, hyperdense on arterial phase and isodense on portovenous phase. Presence of a central low attenuation scar can be seen in 50% but can also present in adeno and hepatocellular carcinoma.

How well did you know this?
1
Not at all
2
3
4
5
Perfectly
3
Q
  1. Which of following causes of diffuse hepatic surface nodularity can be difficult to distinguish from cirrhosis?

A. Chronic Budd-Chiari syndrome

B. Chronic portal vein thrombosis

C. Treated breast cancer metastases to the liver

D. Pseudomyxomaperitonei

E. Miliary metastases

A

C. Treated breast cancer metastases to the liver

Post-chemotherapy changes to hepatic metastases from breast cancer can cause fine diffuse nodularity resembling cirrhosis known as ‘pseudocirrhosis.’

B-D cause coarse nodularity rather than fine and diffuse nodules in cirrhosis. Mets is unlikely to mimic cirrhosis.

How well did you know this?
1
Not at all
2
3
4
5
Perfectly
4
Q
  1. US of a 44-year-old man shows a 7cm cystic lesion in segment 8 of the liver. There is some dependent debris and an apparent cyst within cyst appearance. Which is the most likely diagnosis?

A. Haemangioma

B. Abscess

C. Hydatid disease

D. Hepatic cyst

E. Cystic metastasis

A

C. Hydatid disease

The cyst within cyst appearance represents daughter cyst. Other features include wall calcification.

How well did you know this?
1
Not at all
2
3
4
5
Perfectly
5
Q
  1. A 40-year-old woman has an ultrasound for investigation of gallstones. She is otherwise fit and well. A large lesion is found in the left lobe of the liver. CT is performed for further characterisation pre-contrast, arterial and porto venous phases. The 8cm lesion appears isointense to liver parenchyma on the non-contrast CT uniformly avidly enhancing in the arterial phase and again isointense to liver parenchyma on the portal phase. A low attenuated central scar is noted within the lesion. Which is the most likely diagnosis?

A. Hepatic adenoma

B. FNH

C. Atypical haemangioma

D. Focal fat

E. Fibrolamella carcinoma (FLC)

A

B. FNH

FLC is usually a solidly lobulated well-defined turn, usually low attenuation on unenhanced CT and an even lower attenuation central scar, often with punctate calcification. Delayed enhancement of the scar can occur in both FNH and FLC. However, the scar does not calcify in FNH.

How well did you know this?
1
Not at all
2
3
4
5
Perfectly
6
Q
  1. During discussion with a patient due for Radio-frequency Ablation (RFA) treatment of HCC, she asks you about potential increased risk of tumour seeding along needle track, which she has read about on the internet. Which of the following statements is most true?

A. Lesions with a subscapular location are at lower risk

B. Mortality rates range from 1.5-2%

C. Major complication rates range from 5-10%

D. Minor complication rates typically 10-15%

E. 0.5% risk of tumour seeding down needle track

A

E. 0.5% risk of tumour seeding down needle track

Uncommon and late complication of RFA. In patients with HCC, tumour seeding occurred in 8/1610 cases in a multicenter study and 1/187 in a single series lesions with Subcapsular location. Invasive tumour pattern/poorly differentiated are at higher risk. Mortality rates are typically 0.1-0.5%, major complications 2.2-3.1%, and minor complications 5-8.9%.

How well did you know this?
1
Not at all
2
3
4
5
Perfectly
7
Q
  1. A 40-year-old male presents with an indeterminate liver lesion. He is Hep C positive with previous history of Intravenous Drug Use (IVDU) and was successfully treated for T2NO colorectal cancer last year. Ultrasound demonstrates a 1.5cm slightly hypoechoic lesion in segment 8. This lesion takes up contrast in homogenously in the arterial phase and demonstrates washout in the portovenous and delayed phases, becoming hypoechoic with regards to surrounding liver parenchyma. Which is the most likely diagnosis?

A. Metastasis

B. Focal fatty sparing

C. Capillary haemangioma

D. Hepatic abscess

E. Regenerate nodule

A

A. Metastasis

Rapid washout and residual hypoechoic lesions are features of metastatic deposit.

How well did you know this?
1
Not at all
2
3
4
5
Perfectly
8
Q
  1. Which is the most common MR feature of autoimmune hepatitis?

A. MR is diagnostic test for AIH

B. Surface nodules rarely present

C. PV thrombosis is frequently present

D. Fibrosis better evaluated on CT than MR

E. Enlarged preportal space is a recognised finding

A

E. Enlarged preportal space is a recognised finding

Although AIH has a wide variability in imaging appearances, from normal to end-stage chronic liver disease, the most common feature on CT and MR in one series is surface nodularity.

How well did you know this?
1
Not at all
2
3
4
5
Perfectly
9
Q
  1. Regarding Wilson’s disease:

A. Is autosomal dominant in inheritance

B. Hyperdense nodules on non-enhanced CT is the most common finding

C. Honeycomb pattern is most evident on the pre-contrast study

D. The honeycomb pattern is more evident on T1 than T2

E. Occurs due to overproduction of copper

A

B. Hyperdense nodules on non-enhanced CT is the most common finding

Most cases demonstrate hyperdense nodules on the non-contrast study, some of which became isodense with contrast. A honeycomb appearance is seen in portal and parenchymal phases and on T2 sequences on MR.

How well did you know this?
1
Not at all
2
3
4
5
Perfectly
10
Q
  1. Which of the following is a cause of generalised increase in density pre-iv contrast on CT?

A. Fatty infiltration

B. Malignant infiltration

C. Budd-Chiari

D. Amyloidosis

E. Amiodarone treatment

A

E. Amiodarone treatment

Fatty infiltration, malignant infiltration, Budd-Chiari and amyloidosis all demonstrate generalized low density on pre-contrast CT.

How well did you know this?
1
Not at all
2
3
4
5
Perfectly
11
Q
  1. A 42-year-old woman is being investigated following a new diagnosis of hepatitis C. Which of the following would have the highest accuracy in the characterization of a focal liver lesion in this patient?

A. Ultrasound

B. Doppler ultrasound

C. Microbubble contrast enhanced ultrasound (Ceus )

D. Combined US and AFP

E. Combined US and Doppler US

A

C. Microbubble contrast enhanced ultrasound (Ceus )

CEUS can be used to characterise focal liver lesions with 96.6% accuracy. Small HCC (1-2cm) can be diagnosed due to arterial phase hypervascularity and washout characteristics.

How well did you know this?
1
Not at all
2
3
4
5
Perfectly
12
Q
  1. A 45-year-old man 3 weeks post-op for liver transplantation, develops non-specific abdominal pain with a slight rise in liver function tests. A Doppler ultrasound is requested. Which is the most common vascular complication of orthoptic liver transplantation?

A. Hepatic vein thrombosis

B. Portal vein thrombosis

C. IVC thrombosis

D. Hepatic artery thrombosis

E. Hepatic artery stenosis

A

D. Hepatic artery thrombosis

Hepatic artery thrombosis occurs in 2-12% of cases, usually between 15-132 days post-operatively. The donor celiac axis is anastmosed to the recipient hepatic artery in orthoptic liver transplants. This can be either at the bifurcation of the hepatic into the right and left hepatic arteries, or at the takeoff of the gastroduodenal artery. As the hepatic artery is the only vascular supply to the bile ducts, hepatic artery thrombosis and stenosis can lead to biliary ischemia

How well did you know this?
1
Not at all
2
3
4
5
Perfectly
13
Q
  1. A 34-year-old male visitor to the United Kingdom under the care of the medical team has abnormal liver function tests. US reveals periportal hyperechogenicity and gallbladder wall thickening but no biliary dilatation. The patient has a liver MRI study. T2 images show high signal intensity bands along the portal tracts and note is made of splenomegaly with siderotic nodules. Gadolinium enhanced T1 images confirm enhanced periportal bands. Which of the following is the most likely causative organism?

A. Strongyloides

B. Shistosomiasis

C. Fasciola hepatica flatworm

D. Ascariasis

E. Echinococcus

A

B. Shistosomiasis

Schistosomiasis is the most likely cause. Periportal fibrosis with signs of portal hypertension, splenomegaly and siderotic nodules are typical features of hepatosplenic schistosomiasis

How well did you know this?
1
Not at all
2
3
4
5
Perfectly
14
Q
  1. A 44-year-old man undergoes ultrasound of the abdomen during which the liver is incidentally noted to be of diffusely increased echogenicity, with attenuation of the ultrasound beam and poor visualization of the intrahepatic architecture. Which of the following imaging features is most likely in this condition?

A. liver echogenicity less than that of renal cortex on ultrasound scan

B. relatively hypoattenuated intrahepatic vessels on unenhanced CT

C. liver attenuation 10 HU greater than that of the spleen on unenhanced CT

D. absolute liver attenuation of >40 HU on contrast-enhanced CT

E. loss of liver signal intensity on out-of-phase gradient echo MR images

A

E. loss of liver signal intensity on out-of-phase gradient echo MR images

Fatty liver describes a spectrum of conditions characterized by triglyceride accumulation within hepatocytes. It is common, affecting around 15% of the general population, but is more prevalent among those with obesity, hyperlipidaemia and high alcohol consumption. Fatty liver may be diagnosed on ultrasound scan if liver echogenicity exceeds that of renal cortex, with attenuation of the ultrasound beam, loss of definition of the diaphragm and poor visualization of the intrahepatic architecture. CT featuresinclude absolute attenuation of less than 40 HU on contrast-enhanced CT and, onunenhanced CT, liver attenuation at least 10 HU less than that of spleen, and relativelyhyperattenuating liver vasculature. Chemical shift gradient echo imaging is the mostwidely used MR technique for assessment of fatty liver, demonstrating signal intensityloss on out-of-phase images compared with in-phase images.

How well did you know this?
1
Not at all
2
3
4
5
Perfectly
15
Q
  1. Which venous structure divides the liver into the right and left lobes?

A. right hepatic vein

B. middle hepatic vein

C. left hepatic vein

D. left portal vein

E. right portal vein

A

B. middle hepatic vein

The functional segmental anatomy of the liver is based on the distribution of the three major hepatic veins. The middle hepatic vein divides the liver into left and right lobes. The left hepatic vein divides the left lobe into medial and lateral segments. The right hepatic vein divides the right lobe into anterior and posterior segments. In addition, the four sections are further subdivided in a transverse plane by an imaginary line drawn between the right and left portal veins. Segments run in a clockwise fashion, with segments III, IV(b), V and VI lying below the portal veins and segments VII, VIII, IV(a) and II lying above. The caudate lobe (segment I) lies posterior and to the right of the inferiorvena cava.

How well did you know this?
1
Not at all
2
3
4
5
Perfectly
16
Q
  1. A previously well, 28-year-old man recently returned from the Far East becomes acutely unwell with fever and right upper quadrant pain. Ultrasound scan demonstrates a well-defined, rounded, 7 cm hypoechoic lesion in the right lobe of the liver contiguous with the liver capsule, with fine homogeneous, low-level internal echoes and acoustic enhancement. What is the most likely diagnosis?

A. pyogenic abscess

B. amoebic abscess

C. fungal abscess

D. hydatid disease

E. incidental simple hepatic cyst

A

B. amoebic abscess

Pyogenic abscesses are the commonest type of liver abscess in developed countries, and are most frequently due to ascending cholangitis from benign or malignant obstructive biliary disease. They are often poorly defined with irregular walls on ultrasound scan,and may contain debris or demonstrate intense hyperechogenicity when containing gas. Amoebic abscesses tend to occur in younger, more acutely unwell patients from high prevalence areas or with a history of recent travel. They are treated medically whereas pyogenic abscesses usually require percutaneous or surgical drainage. Fungal abscesses are usually multiple and occur in immunosuppressed individuals. Hydatid disease tends to be asymptomatic or present with biliary colic. Characteristic ultrasound scan features include daughter cysts and detachment of the endocyst, giving rise to ‘floating membranes’ within the cyst cavity.

How well did you know this?
1
Not at all
2
3
4
5
Perfectly
17
Q
  1. A 43-year-old woman is incidentally found to have a well-defined, rounded, low-density, 2 cm lesion in the liver on unenhanced CT. Contrast-enhanced CT demonstrates peripheral nodular arterial enhancement with complete fill-in on delayed images. What is the most likely diagnosis?

A. hepatic haemangioma

B. hepatocellular carcinoma

C. simple hepatic cyst

D. focal fatty infiltration

E. focal nodular hyperplasia

A

A. hepatic haemangioma

Haemangiomas are the most common benign liver tumour. They are often asymptomatic but may present with hepatomegaly or rarely spontaneous haemorrhage. Typical CT features of a hepatic haemangioma are of a well-defined hypodense mass on unenhanced CT, with early peripheral enhancement after intravenous contrast followed by complete fill-in on delayed images. Hepatocellular carcinoma is seen as a hypodense mass and usually demonstrates contrast enhancement during the arterial phase, but enhancement decreases on delayed images. Focal fatty infiltration usually has ageographic distribution, and, like simple hepatic cysts, does not demonstrate contrast enhancement. Focal nodular hyperplasia is usually isodense on unenhanced CT and, although it tends to show intense transient arterial phase enhancement, is often isodense during the portal phase. A central scar, if present, may demonstrateenhancement on delayed images.

How well did you know this?
1
Not at all
2
3
4
5
Perfectly
18
Q
  1. A 68-year-old man presents with acute abdominal pain. As well as other pathology, CT of the abdomen reveals multiple linear branching structures with an attenuation value of –1000 HU in the liver extending to the periphery. What are these appearances most likely to represent?

A. gas in the portal venous system

B. gas in the biliary tree

C. portal venous thrombosis

D. intrahepatic biliary dilatation

E. fatty infiltration of the liver

A

A. gas in the portal venous system

Portal venous gas is identified on CT as linear branching structures of air density extending to the periphery of the liver, presumably due to the direction of portal venous flow. The commonest cause in adults is mesenteric infarction, when it is a poor prognostic sign. In infants the commonest cause is necrotizing enterocolitis, when it does not necessarily imply a poor outcome. In contrast, gas within the biliary tree is central and does not extend into the peripheral 2 cm of the liver. In portal venous thrombosis, a focal hypodensity is seen within the portal vein on contrast-enhanced CT. Intrahepatic biliary dilatation appears as dilated branching ductal structures of fluid density. In fatty infiltration of the liver, vessels may appear hyperattenuating on unenhanced CT in contrast to the hypodense liver.

How well did you know this?
1
Not at all
2
3
4
5
Perfectly
19
Q
  1. A 30-year-old woman on the oral contraceptive pill undergoes unenhanced CT of the abdomen, which demonstrates a well-circumscribed, slightly hypoattenuating mass in the liver. Which additional radiological finding would favour a diagnosis of hepatic adenoma rather than focal nodular hyperplasia?

A. measured lesion size of 3 cm

B. accompanying acute subcapsular haematoma

C. transient arterial-phase enhancement

D. normal uptake on 99mTc-labelled sulphur colloid scan

E. hypodense central stellate scar

A

B. accompanying acute subcapsular haematoma

Focal nodular hyperplasia (FNH) is a benign hamartomatous malformation commonest in young women. Lesions are usually smaller than 5 cm, and contain a central stellate scar in up to one-third of cases. Hepatic adenomas are benign tumours averaging 8–10 cm in size, seen predominantly in young women and related to oral contraceptive use. Lesions have a propensity for spontaneous haemorrhage, presenting as subcapsular haematoma or haemoperitoneum. FNH, though highly vascular, rarely undergoes spontaneous haemorrhage. FNH usually contains sufficient functioning Kupffer cells to demonstrate normal or increased uptake on 99 mTc-labelled sulphur colloid scan, whereas hepatic adenoma, composed of hepatocytes and non-functioning Kupffer cells, appears as a focal photopenic lesion. Both lesions demonstrate transient arterial enhancement following intravenous contrast.

How well did you know this?
1
Not at all
2
3
4
5
Perfectly
20
Q
  1. A 45-year-old patient with cirrhosis is found to have a focal liver lesion on ultrasound scan, clinically suspected to be hepatocellular carcinoma. What would be the expected appearances of the lesion on T2W MR images following infusion of superparamagnetic iron oxide particles?

A. increased signal intensity compared with rest of liver

B. decreased signal intensity compared with rest of liver

C. lesion signal intensity unchanged; rest of liver increased signal intensity

D. lesion signal intensity unchanged; rest of liver decreased signal intensity

E. no effect on appearances on T2W images

A

D. lesion signal intensity unchanged; rest of liver decreased signal intensity

Superparamagnetic iron oxide (SPIO) particles are iron-based particles of 30–150 nm, which, when administered as an infusion 1–4 hours prior to imaging, act as a negative MR contrast agent. They target the reticuloendothelial system, being taken up by macrophages throughout the body, but are preferentially accumulated by the Kupffer cells of the liver. Their superparamagnetic properties result in T2 and T2shortening of the tissues that accumulate the particles, which show reduced signal intensity on T2W, T2W and, to a lesser extent, T1W images. Most liver tumours do not exhibit uptake, as they are deficient in Kupffer cells. However, as the rest of the liver accumulates SPIO and darkens preferentially, the tumour appears of increased conspicuity. SPIO particles are particularly used, in combination with gadolinium, to improve detection of hepatocellular carcinoma in cirrhotic patients, in whom the parenchymal changes of fibrosis and regenerative nodules make detection with gadolinium alone difficult.

How well did you know this?
1
Not at all
2
3
4
5
Perfectly
21
Q
  1. A 35-year-old woman on the oral contraceptive pill presents with right upper quadrant pain, shortness of breath and leg oedema. Ultrasound scan of the abdomen demonstrates hepatosplenomegaly and ascites. The hepatic veins are not visualized on Doppler ultrasound scan. What is the most likely diagnosis?

A. acute Budd–Chiari syndrome

B. primary biliary cirrhosis

C. passive hepatic congestion

D. hepatic veno-occlusive disease

E. viral hepatitis

A

The answer is: A

Budd–Chiari syndrome is caused by obstruction of hepatic venous outflow, which may, in turn, be caused by membranous obstruction of the suprahepatic IVC by a congenital web, or hepatic venous thrombosis due to hypercoagulable state, tumour or trauma. Patients develop hepatosplenomegaly and intractable ascites. Doppler ultrasound scan demonstrates non-visualization of, or thrombus within, one or more hepatic veins. CT findings reflect severely impaired blood flow to the liver, with a ‘flip-flop’ enhancement pattern after contrast administration. Early images show prominent central liver enhancement with poor peripheral enhancement, whereas delayed images show central washout with peripheral enhancement. The caudate lobe is typically spared because of its separate venous drainage directly into the IVC, and enhances normally. Passive hepatic congestion complicates heart failure, and results in distended hepatic veins and IVC. Hepatic veno-occlusive disease refers to occlusion of small centrilobular hepatic veins following radio- and chemotherapy in bone-marrow transplant recipients, or related to alkaloid consumption. The main hepatic veins and IVC are normal. Hepaticvenous involvement is not a feature of viral hepatitis or primary biliary cirrhosis.

How well did you know this?
1
Not at all
2
3
4
5
Perfectly
22
Q
  1. An 86-year-old, otherwise well woman is admitted with abdominal pain and undergoes plain abdominal radiography. This demonstrates a normal bowel gas pattern, but the liver and spleen are noted to be of increased density with a stippled appearance. What is the most likely cause?

A. haemochromatosis

B. thorotrastosis

C. amiodarone therapy

D. sickle cell anaemia

E. glycogen storage disease

A

B. thorotrastosis

Thorotrast (thorium dioxide), an alpha-emitting radioactive isotope of atomic number 90and long half-life, was used as a contrast agent until the mid-1950s, predominantly for cerebral angiography and reticuloendothelial imaging. It is retained indefinitely by the reticuloendothelial system, and results in increased density of the liver, spleen and lymph nodes with a characteristic stippled appearance. It is associated with delayed malignancies, including angiosarcoma, cholangiocarcinoma and hepatocellular carcinoma. Haemochromatosis may result in diffusely increased density of the liver and spleen, but usually presents earlier in life. Amiodarone may result in increased liver attenuation of 95–145 HU (normal 30–70 HU), but splenic involvement is not usually a feature. Sickle cell anaemia can result in a shrunken calcified spleen, but again is unlikely in this age group. Glycogen storage disease can result in a generalized increase or decrease in hepatic density on CT, but increased splenic density is not a feature.

How well did you know this?
1
Not at all
2
3
4
5
Perfectly
23
Q
  1. A 51-year-old man with alcoholic cirrhosis presents with jaundice. CT of the abdomen reveals an encapsulated, 20 mm focal area of low density in the liver, which demonstrates arterial-phase enhancement and rapid washout on delayed imaging. What is the most likely diagnosis?

A. regenerative nodule

B. dysplastic nodule

C. hepatocellular carcinoma

D. hepatic haemangioma

E. focal fatty sparing

A

C. hepatocellular carcinoma

Nodules are a common finding in cirrhosis, and differentiation of benign nodules from hepatocellular carcinoma (occurring in 7–12% of patients) is vital. Most nodules are regenerative nodules, representing reparative attempts by hepatocytes in response to liver injury. These are typically under 10 mm in size and appear isodense to liver parenchyma on CT, unless they contain iron deposits (siderotic nodules), in which case they may be slightly hyperdense. Dysplastic nodules are proliferative premalignant lesions found in 15–25% of cirrhotic livers. They resemble regenerative nodules on CT but are usually larger than 10 mm. Hepatocellular carcinomas usually appear as encapsulated hypodense masses that demonstrate rapid arterial enhancement and early washout of contrast on delayed images. Hepatic haemangioma usually appears as a low density mass, but has different enhancement characteristics, demonstrating peripheral enhancement with complete fill-in on delayed images. Focal fatty sparing appears as an area of normal density in a generally hypodense liver and does not demonstrate contrast enhancement.

How well did you know this?
1
Not at all
2
3
4
5
Perfectly
24
Q
  1. A 27-year-old woman presents with upper abdominal pain and is found to have a palpable right upper quadrant mass on examination. CT demonstrates a low attenuation lesion in the right lobe of the liver. Which imaging feature would favour a diagnosis of fibrolamellar carcinoma of the liver rather than focal nodular hyperplasia?

A. calcifications within a central scar

B. lesion size of 3 cm

C. multiple lesions

D. increased uptake on sulphur colloid scan

E. central scar hyperintense on T2W images

A

A. calcifications within a central scar

Fibrolamellar carcinoma of the liver is an uncommon variant of hepatocellular carcinoma, typically presenting as a large, 5-20 cm liver mass in a young patient with no risk factors. Typical features are of an encapsulated mass with a prominent central fibrous scar. The scar often contains areas of calcification, and appears hypointense on T1- and T2-weighted images. Focal nodular hyperplasia (FNH) is a hamartomatous malformation also most commonly seen in young woman. However, lesion size is usually <5 cm and, although a central fibrous scar is also a common feature, this usually appears hyperintense on T2-weighted images due to vascular channels and oedema, and calcifications within it are extremely rare. Both pathologies may result in multiple lesions, with FNH being multiple in 20% and fibrolamellar carcinoma demonstrating satellite lesions in 10–15%. FNH is the only liver lesion with sufficient Kupffer cells to cause normal or increased uptake on sulphur colloid scan.

How well did you know this?
1
Not at all
2
3
4
5
Perfectly
25
Q
  1. 45-year-old man undergoes ultrasound scan of the abdomen 2 days following orthotopic liver transplantation, which demonstrates periportal oedema and a small fluid collection at the hilum of the liver. What is the most likely diagnosis?

A. graft rejection

B. hepatic arterial thrombosis

C. portal vein stenosis

D. bile leak

E. normal post-transplantation findings

A

E. normal post-transplantation findings

Orthotopic liver transplantation is the treatment of choice for patients with end-stage liver disease for which no other therapy is available. Surgery involves one arterial anastomosis (hepatic artery), at least two venous anastomoses (portal vein and IVC) and a biliary anastomosis, and complications may occur at any of these sites. Vascular complications are the most frequent cause of graft loss, and most commonly involve the hepatic artery, with portal venous and IVC complications being relatively infrequent. Biliary complications occur in up to 34% of cases, and are the second most common cause of liver dysfunction after graft rejection. They include leak, stricture and obstruction. Other complications include fluid collections, infection and malignancy. Normal findings following liver transplantation include a small amount of free intraperitoneal fluid in the perihepatic region, especially at the hilum, or in the fissure for the ligamentum teres, which usually resolves within a few weeks. Other normal findings are a right fluid pleural effusion, and periportal oedema, attributed to lymphatic cchannel dilatation due to lack of normal lymphatic drainage.

How well did you know this?
1
Not at all
2
3
4
5
Perfectly
26
Q
  1. A 40-year-old man with hyperpigmentation, arthalgia and diabetes mellitus is clinically suspected to have primary haemochromatosis. What are the most likely findings on liver MRI in this condition?

A. normal appearances of the liver

B. decreased signal intensity on T1W and T2Wimages

C. decreased signal intensity on T1W and increased signal intensity on T2W images

D. increased signal intensity on T1W and decreased signal intensity on T2W images

E. increased signal intensity on T1W and T2WImages

A

B. decreased signal intensity on T1W and T2Wimages

In primary haemochromatosis, there is increased duodenal absorption and parenchymal retention of dietary iron, which is accumulated within the liver, pancreas, heart and pituitary gland. Intracellular iron deposits within hepatocytes result in a paramagnetic susceptibility effect, leading to marked shortening of T1 and T2 relaxation times of adjacent protons. This manifests as a marked reduction in liver signal intensity on T2Wand T2*W images, and a moderate loss of signal intensity on T1W images.

How well did you know this?
1
Not at all
2
3
4
5
Perfectly
27
Q
  1. A 54-year-old woman undergoes CT of the abdomen and pelvis for weight loss and is found to have multiple, irregular, calcified, low-attenuation lesions in the liver, suggestive of metastases. What is the most likely primary lesion?

A. invasive ductal carcinoma of the breast

B. mucinous adenocarcinoma of the gastrointestinal tract

C. osteosarcoma

D. non-small-cell lung carcinoma

E. carcinoid

A

B. mucinous adenocarcinoma of the gastrointestinal tract

Calcified liver metastases represent up to 3% of liver metastases, and are most commonly seen with mucinous carcinomas of the gastrointestinal tract. They are also seen in osteosarcoma, breast cancer, lung cancer and carcinoid, but these are less common.

How well did you know this?
1
Not at all
2
3
4
5
Perfectly
28
Q
  1. A 45-year-old woman undergoes abdominal ultrasound scan. The portal vein measures 16 mm in diameter and demonstrates continuous monophasic flow without respiratory variation. Portal vein flow velocity is hepatopetal and is measured to be 7 cm/s. What is the most likely diagnosis?

A. normal findings

B. Budd–Chiari syndrome

C. portal hypertension

D. cavernous transformation of the portal vein

E. portal vein thrombosis

A

C. portal hypertension

The normal portal vein measures up to 13 mm in diameter when measured in the AP direction where the portal vein crosses the inferior vena cava during quiet respiration in a supine patient. Portal venous flow is normally 12–30 cm/s and demonstrates respiratory variation but little or no pulsatility, though this may be seen in thin patients. Normal flow is hepatopetal (anterograde flow into the liver). Portal hypertension is defined as an increase in portal venous pressure above10 mmHg, and is most commonly caused by cirrhosis in the western world. As portal pressure increases, portal vein diameter increases, and portal flow loses its respiratory fluctuation and becomes slow and turbulent. Reversed (hepatofugal) flow may occur in 8% of patients and is generally associated with a reduced portal vein diameter. Other findings include portosystemic collaterals, splenomegaly and ascites. In portal vein thrombosis, portal vein diameter increases, but no flow is seen on Doppler ultrasound scan. Echogenic thrombus may be seen within the lumen. Cavernoustrans formation of the portal vein may occur with chronic portal vein thrombosis, representing a conglomerate of collateral veins. Budd–Chiari syndrome affects the hepatic veins.

How well did you know this?
1
Not at all
2
3
4
5
Perfectly
29
Q
  1. A 26-year-old man with AIDS presents with weight loss. He is noted to have multiple raised purple skin lesions on examination. Contrast-enhanced CT of the abdomen and pelvis demonstrates multiple, subcentimetre, low-attenuation nodules within the liver, as well as high attenuation lymphadenopathy at the porta hepatis, retrocaval and aortocaval regions. What is the most likely diagnosis?

A. fungal infection

B. multiple haemangiomas

C. lymphoma

D. Kaposi’s sarcoma

E. mycobacterial disease

A

D. Kaposi’s sarcoma

Kaposi’s sarcoma is a low-grade tumour of the blood and lymphatic vessels that primarily affects the skin but may cause disseminated disease in other organs. It is an AIDS-defining illness and is the most common AIDS-related neoplasm. The commonest manifestation is of multiple raised purplish skin lesions, but lymphadenopathy is the second commonest feature in AIDS-related Kaposi’s sarcoma. Typical appearances are of abdominopelvic lymph nodes that enhance after intravenous contrast due to high vascularity, appearing to be of higher attenuation than skeletal muscle. Liver involvement occurs in 34% of cases at autopsy, and typically causes multiple 5–12 mm nodules that are hyperechoic on ultrasound scan and of low attenuation on CT. Skin lesions are present in most cases, and help to distinguish Kaposi’s sarcoma from other conditions such as fungal microabscesses and multiple haemangiomas, which may have similar appearances on CT. Mycobacterial disease is characteristically associated with low-attenuation lymphadenopathy. Non-Hodgkin’s lymphoma is the second commonest AIDS-related neoplasm, and may cause multiple low-attenuation liver lesions, but it is not associated with skin lesions or high-attenuation lymphadenopathy.

How well did you know this?
1
Not at all
2
3
4
5
Perfectly
30
Q
  1. A 45-year-old woman undergoes a follow-up staging CT of the chest, abdomen and pelvis after treatment for metastatic breast cancer. Compared with her initial staging scan, there is a generalized decrease in the attenuation value of the liver. No focal liver lesion or other new feature is seen. What is the most likely cause?

A. diffuse metastatic disease

B. fatty liver related to chemotherapy

C. hepatic venous congestion

D. amyloidosis

E. Budd–Chiari syndrome

A

B. fatty liver related to chemotherapy

Chemotherapeutic agents are commonly associated with fatty liver. Diffuse fat deposition (the commonest pattern) causes a generalized decrease in the attenuation value of the liver on CT, and may be diagnosed with an absolute liver attenuation value of less than 40 HU on contrast-enhanced CT. It may also be diagnosed at unenhanced CT if the liver attenuation value is at least 10 HU less than that of the spleen. Liver metastases usually present as focal, low-attenuation lesions on portal phase imaging. Hepatic venous congestion causes a diffuse decrease in attenuation but is associated with enlargement of the inferior vena cava and hepatic veins due to elevated central venous pressure. Amyloidosis can cause a generalized decrease in liver attenuation, but more commonly appears as discrete areas of low attenuation with reduced contrast enhancement. Budd–Chiari syndrome may also result in a diffuse decrease in liver attenuation, but there is usually patchy liver enhancement and poor visualization of the hepatic veins.

How well did you know this?
1
Not at all
2
3
4
5
Perfectly
31
Q

6 A 61-year-old has alcohol-related chronic liver disease. Which of the conditions is he not at increased risk of developing compared the general population?

(a) Non-specific interstitial pneumonitis

(b) Bacterial pneumonia

(c) Hydrothorax

(d) Pulmonary hypertension

(e) Acute respiratory distress syndrome

A

(a) Non-specific interstitial pneumonitis

There is no increased risk of NSIP or any other interstitial lung disease as a result of cirrhosis. Patients with cirrhosis have altered immunity and undergo changes to the vascular bed both the liver and the lungs.

How well did you know this?
1
Not at all
2
3
4
5
Perfectly
32
Q

17 An unenhanced CT of the liver is performed. The liver has density of 60 HU and the spleen has a density of 50 HU. How might you account for these findings?

(a) Normal findings

(b) Diffuse fatty infiltration

(c) Haemochrornatosis

(d) Wilson’s disease

(e) Budd-Chiari syndrome

A

(a) Normal findings

The normal liver has a density of 50-70 HU. Fatty infiltration will reduce this as the atomic numbers of the elements C, H and O. Iron and copper deposition can raise the density, as they have high atomic numbers.

How well did you know this?
1
Not at all
2
3
4
5
Perfectly
33
Q

19 A 50-year-old man undergoes liver transplantation. A routine liver US comments that the intra-hepatic arterial Doppler has a low resistance index (0.48) and a prolonged systolic acceleration time. What diagnosis does this suggest?

(a) Hepatic artery stenosis

(b) Portal vein occlusion

(c) Hepatic artery pseudoaneurysm

(d) Graft rejection

(e) Hepatic artery thrombosis

A

(a) Hepatic artery stenosis

The tardus et parvus waveform described here is seen distal to a stenosis. At the stenosis, a jet phenomenon may be seen with greatly increased flow.

How well did you know this?
1
Not at all
2
3
4
5
Perfectly
34
Q

Hepatocellular adenomas are not associated with which of the following?

(a) Spontaneous rupture

(b) Oral contraceptive pill

(c) Hepatocellular carcinoma

(d) Androgenic steroids

(e) Cholangiocarcinoma

A

(e) Cholangiocarcinoma

Hepatocellular adenomas are uncommon benign tumours which comprise hepatocytes with no portal tracts or bile ducts. They are associated with the oral contraceptive pill and androgenic steroids. On imaging, they may have a scar or a pseudocapsule and enhance avidly in the arterial phase which can make differentiation from FNH difficult. There is a tendency to bleed or rupture, 1 % are thought to transform in to malignant lesions (HCC).

How well did you know this?
1
Not at all
2
3
4
5
Perfectly
35
Q

A 28-year-old man with cystic fibrosis and abnormal liver function is referred to you for liver imaging by his clinical team. Which of the following are not associated with cystic fibrosis?

(a) Nodular regenerative hyperplasia

(b) Steatosis

(c) Sclerosing cholangitis

(d) Cirrhosis

(e) Cholelithiasis

A

(a) Nodular regenerative hyperplasia

Cystic fibrosis produces a range of abnormalities, which may lead to fibrosis, cirrhosis and portal hypertension. A micro-gallbladder is also seen commonly.

How well did you know this?
1
Not at all
2
3
4
5
Perfectly
36
Q

2 A 42-year-old presents acute right upper quadrant pain and abdominal distension. She has a 1-week history of dark urine, particularly in the mornings, and is found to have pancytopenia. What is the most likely diagnosis?

(a) Acute cholecystitis

(b) Acute pyelonephritis

(c) Bud-Chiari syndrome

(d) Ruptured renal cell carcinoma

(e) Infectious mononucleosis

A

2 (c)

The patient has classical features of underlying paroxysmal nocturnal haemoglobinuria (PHH), which predisposes to Budd-Chiari syndrorme. Budd-Chiari syndrome presents with these symptoms; the severity of the initial liver disease is variable depending upon the extent of venous occlusion.

How well did you know this?
1
Not at all
2
3
4
5
Perfectly
37
Q

3 An MRI liver report reads: ‘There is a hyperintense lesion on T2-weighted imaging, which is hypointense on TI -weighted images. administration of i.v. Gadolinium, there is peripheral nodular enhancement in the arterial phase progressive enhancement on the portal venous and delayed phases.” What is the most likely diagnosis?

(a) Hemangioma

(b) Cyst

(c) Focal nodular hyperplasia

(d) Hepatocellular adenoma

(e) Hepatocellular carcinoma

A

(a) Hemangioma

These are characteristic findings of a haemangioma. A cyst will enhance; and adenoma are benign hyper vascular lesions that equilibrate in the portal phase. HCC is hypervascular with washout on delayed imaging; the liver is usually cirrhotic.

How well did you know this?
1
Not at all
2
3
4
5
Perfectly
38
Q

5 You are asked to review a CT for a patient with a metastatic melanoma in a clinical trial. There is a single deposit within the liver which measured 10 cm on the initial study and now measures 6 cm. How should you classify the response by RECIST criteria?

(a) Complete Response

(b) Complete Response Unconfirmed

(c) Partial Response

(d) Stable Disease

(e) Progressive Disease

A

(c) Partial Response

A reduction of > 30%, falling short of a complete response, constitutes a partial response.

How well did you know this?
1
Not at all
2
3
4
5
Perfectly
39
Q

13 A 33-year-old woman undergoes an MRI study to characterise a 4 cm focal liver lesion found incidentally on US. The lesion is isointense to liver on TI and T2 weighted images, enhances homogeneously and avidly in the arterial phase and is isointense in the portal and delayed phases. 1 hour after the administration of contrast mediun (Gd-Bopta), the lesion is hyperintense to liver. What is the most likely diagnosis?

(a) Hemangioma

(b) Focal nodular hyperplasia

(c) Hepatocellular adenoma

(d) Fibrolamellar hepatocellular carcinoma

(e) Metastasis

A

(b) Focal nodular hyperplasia

These imaging features are characteristic of FNH. FNH is a tumour thought to represent a hyperplastic response to a pre-existing arterial malformation. It is most commonly seen in women and has no malignant potential. A central scar may be seen which shows delayed enhancement.

How well did you know this?
1
Not at all
2
3
4
5
Perfectly
40
Q

14 A 52-year woman with a history of atrial fibrillation and rheumatoid arthritis presents with right-sided renal colic. On the unenhanced CT study there is diffuse low density in the liver. Which of the following right account for this appearance?

(a) Amyloidosis

(b) Amiodarone therapy

(c) Haemochromatosis

(d) Haemosiderosis

(e) Wilson’s disease

A

(a) Amyloidosis

The most common cause of diffuse low attenuation is fatty infiltration; amyloidosis is another due to amyloid deposition. Amyloidosis can be a primary condition, or secondary to chronic infectious, chronic inflammatory disease (e.g. rheumatoid arthritis), or multiple myeloma. Answers (b) - (d) increase copper/iron deposition within the liver, which will lead to increased liver density on a pre-contrast CT.

How well did you know this?
1
Not at all
2
3
4
5
Perfectly
41
Q

16 A 47-year old man presents with atypical RIF pain; a CT abdomen is requested to rule out appendicitis. No cause for abdominal pain is identified. The only finding of note is 8 mm low-attenuation lesion within the left lobe of the liver. The lesion lies inferior to the portal vein and lateral to the left hepatic vein. What segment of the liver is the lesion in?

(a) Segment I

(b) Segment II

(c) Segment III

(d) Segment IV-A

(e) Segment IV-B

A

(c) Segment III

The Couinaud classification divides the liver into 8 functionally independent segments, each with its own blood supply and biliary drainage. The portal vein divides the liver into superior and inferior segments. The middle hepatic vein divides the left (segments I-IV ) and right lobes (segments V-VIII). The right hepatic vein divides the right lobe into anterior (V and VIII) and posterior (VI and VII) segments. The left hepatic vein divides the left lobe into medial (segments IV-A and IV-B) and lateral part (segments II and III); segment I is the caudate lobe, situated posteriorly.

How well did you know this?
1
Not at all
2
3
4
5
Perfectly
42
Q

24 A 52-year-old lady undergoes liver transplantation for autoimmune liver disease. 6 months later, she presents with deranged liver function and an MRCP demonstrates an anastomotic stricture. What is the most common aetiology of non-anastomotic strictures in this group?

(a) Ischaemia

(b) Rejection

(c) Recurrent autoimmune liver disease

(d) Biliary cast syndrome

(e) Post-transplantation lymphoproliferative disease

A

(a) Ischaemia

Ischaemia is the underlying cause in approximately 50% of cases; the bile ducts are supplied by the hepatic artery and their blood supply is inevitably disrupted to some extent during transplantation. Evaluation of the hepatic arterial supply to the graft is crucial as thrombosis usually requires re-transplantation in the adult population.

How well did you know this?
1
Not at all
2
3
4
5
Perfectly
43
Q

26 A 56-year-old woman is referred for an US of the liver which shows a solitary 3 cm hypoechoic lesion. US contrast is given which demonstrates the lesion to be hyporeflective with rim enhancement. The rim enhancement fades in the portal venous phase and the lesion becomes increasingly hyporeflective and well-defined. What is the most likely diagnosis?

(a) Cyst

(b) Hemangioma

(c) Focal nodular hyperplasia

(d) Hepatocellular carcinoma

(e) Metastasis

A

(e) Metastasis

These are the characteristic features of a hypovascular metastasis. Lesions enhance in a fashion similar to that seen on CT or MRI.

How well did you know this?
1
Not at all
2
3
4
5
Perfectly
44
Q

27 A 47-year-old man with multifocal hepatocellular carcinoma and chronic liver disease is referred for consideration of liver transplantation. Which of the following would be considered for transplantation in the UK?

(a) 2 lesions measuring 5 cm or less

(b) 2 lesions measuring 4 cm or less

(c) 3 lesions measuring 3 cm or less

(d) 4 lesions measuring 2 cm or less

(e) 5 lesions measuring 2 cm or less

A

(c) 3 lesions measuring 3 cm or less

Transplantation is considered in patients with 1 lesion <5 cm or up to 3 lesions measuring 3 cm or less, the ‘Milan Criteria’.

How well did you know this?
1
Not at all
2
3
4
5
Perfectly
45
Q

41 You are referred a patient with a history of previous left hepatectomy. What liver segments will have been resected?

(a) I &II

(b) II &III

(c) I, II &III

(d) II, III &IV

(e) I, II, III, IV

A

(b) II &III

These segments are also referred to as the left lateral lobe. Resection of segment IV also is termed an extended left hepatectomy. Resection of V, VI, VII and VIII is a right hepatectomy. An extended right hepatectomy includes segment IV also. The caudate lobe is usually only resected during liver transplantation.

How well did you know this?
1
Not at all
2
3
4
5
Perfectly
46
Q

46 With regard to non-alcoholic steato-hepatitis (NASH), which of the statements is not true?

(a) It is seen in up to 30 % of the population

(b) Diabetes mellitus is a risk factor

(c) up to 15% of patient’s progress

(d) Is associated with hepatic fibrosis

(e) May resolve with dietary modifications alone

A

(a) It is seen in up to 30 % of the population

Hepatic steatosis is seen in this proportion of the population; of these, 6-8% progress to NASH, where there is inflammation of the hepatocytes and abnormal liver function.

How well did you know this?
1
Not at all
2
3
4
5
Perfectly
47
Q

58 A 48-year-old man with diabetes mellitus and abnormal liver function undergoes an MRI of the liver. The liver parenchyma has a smooth contour and is relatively hypointense on the TI -weighted image and hypointense to muscle on the T 2- weighted image. What is the most likely diagnosis?

(a) Acute hepatitis

(b) Fatty liver

(c) Haemochromatosis

(d) Autoimmune liver disease

(e) Wilson’s disease

A

(c) Haemochromatosis

The low signal within the liver parenchyma on T2-weighted images is due to the susceptibility artefact from iron overload. Patients with haemochromatosis deposit iron within the skin, heart, liver and pancreas. This degree of iron within the liver can be quantified using MRI

How well did you know this?
1
Not at all
2
3
4
5
Perfectly
48
Q

A previously well 56-year-old man presents with right upper quadrant pain and tenderness associated with nausea and vomiting. A liver US demonstrates the presence of a large cyst containing a number of smaller cysts with a honeycomb appearance. What is the most likely diagnosis?

(a) Cystadenoma

(b) Cholangiocarcinoma

(c) Hydatid cyst

(d) Hepatocellular carcinoma

(e) Caroli disease

A

(c) Hydatid cyst

Hydatid infection is caused by Echinococcus multilocularis, and may give rise to a number of appearances at US with cysts being uni- or multi-loculated, thin or thick-walled, with or without calcification. The appearance described here is referred to as daughter cysts. Cystadenoma gives rise to a large unilocular cyst or septated cyst. Caroli disease is due to duct abnormalities. Hepatocellular carcinoma and cholangiocarcinoma are solid lesions.

How well did you know this?
1
Not at all
2
3
4
5
Perfectly
49
Q

60 Which of the following is not a recognised association of polycystic liver disease?

(a) Non-Hodgkin lymphoma

(b) Hemorrhage

(c) Portal hypertension

(d) Infection

(e) Polycystic kidney disease

A

(a) Non-Hodgkin lymphoma

Polycystic liver disease is a hereditary condition that may or may be associated with polycystic kidney disease. There is malignant potential within the cysts but complications may arise from the sheer size of the cysts, and therapeutic interventions. Aspiration, fenestration and enucleation may provide short term relief, but some cases will require repeated therapy or transplantation.

How well did you know this?
1
Not at all
2
3
4
5
Perfectly
50
Q

Which of the following conditions is not associated with nodular regenerative hyperplasia?

(a) Polycythaemia rubravera

(b) Rheumatoid arthritis

(c) Cirrhosis

(d) Systemic lupus erythematosus

(e) Non-Hodgkin’s lymphoma

A

(c) Cirrhosis

NRH is diffuse nodularity of the liver produced by regenerative nodules in the absence of hepatic fibrosis. Although a rare entity, it is commonly associated with portal hypertension (50% cases).

How well did you know this?
1
Not at all
2
3
4
5
Perfectly
51
Q

Following solid organ transplantation, post-transplantation lymphoproliferative disorder most commonly affects which of the following organs?

(a) Lungs

(b) Liver

(c) Spleen

(d) Kidney

(e) Bowel

A

66 (b)

The liver is affected most frequently, with involvement in up to

45% of liver transplant PTLDs,
40% of pancreas transplant PTLDs
23% of heart transplant PTLDs
0% of lung transplant PTLDs

How well did you know this?
1
Not at all
2
3
4
5
Perfectly
52
Q
  1. An 83 year old woman is investigated for weight loss, and undergoes contrast enhanced CT scan of the chest, abdomen and pelvis. Multiple hypervascular metastases are found in the liver. Which one of the following is most likely to be the primary tumour?

a. Adenocarcinoma of the stomach

b. Invasive ductal carcinoma of the breast

c. Carcinoid tumour

d. Adenocarcinoma of the sigmoid

e. Pancreatic ductal adenocarcinoma

A
  1. c. Carcinoid tumour

Of the options listed, carcinoid tumour is the only primary tumour that typically causes hypervascular liver metastases. Other causes of hypervascular liver metastases are pancreatic islet cell tumours, phaeochromocytoma and renal cell carcinoma. Stomach, breast, lung and colon cancers are associated with hypovascular liver metastases. Liver metastases from carcinoid tumours are more common with increasing size of the primary tumour. The incidence of metastases depends on the location of the primary tumour, where approximately 30% of carcinoids of the ileum metastasise compared to less than 5% of carcinoids of the appendix.

How well did you know this?
1
Not at all
2
3
4
5
Perfectly
53
Q
  1. An asymptomatic 46 year old woman has an MR liver following an incidental finding of a focal mass in the right lobe of the liver on ultrasound. The MR shows an 8 cm isolated lesion. It is high signal on T1-weighted sequences and isointense on T2-weighted sequences relative to the normal liver parenchyma. The lesion is most likely to be which one of the following?

a. Hepatocellular carcinoma

b. Liver metastasis

c. Haemangioma

d. Fibronodular hyperplasia

e. Adenoma

A
  1. e. Adenoma

The lesion is most likely to be a hepatic adenoma. None of the other diagnoses typically share these imaging characteristics. Adenomas are benign growths of hepatocytes and are most commonly seen in young women, particularly associated with oral contraceptive use. Eighty per cent are solitary and found in the right lobe of the liver. The high signal on T1-weighted sequences is due to the presence of fat and/or haemorrhage and can distinguish between this and many other lesions in the liver which tend to be of low T1 signal on MR (e.g. metastases, HCC, haemangiomas and FNH). Occasionally, imaging features can overlap with FNH and the two lesions can be difficult to distinguish. However, the majority of FNH lesions are less than 5 cm in size, whereas adenomas tend to be larger.

54
Q
  1. A 26 year old female has an ultrasound scan for right upper quadrant pain and a heterogenous 5 cm solitary liver lesion with central calcifications, and a hyperechoic scar is seen. Blood tests reveal a negative alpha-fetoprotein. MR shows the lesion is hypointense on T1 and hyperintense on T2-weighted imaging. The central scar is hypointense on both sequences. Which of the following diagnoses is most likely?

a. Hepatic lymphoma

b. Hepatocellular carcinoma

c. Fibrolamellar carcinoma

d. Hepatoblastoma

e. Hepatic angiosarcoma

A
  1. c. Fibrolamellar carcinoma

Fibrolamellar carcinoma occurs in young adults in the absence of normal risk factors for hepatocellular carcinoma. On ultrasound, fibrolamellar carcinoma is of mixed or increased echogenicity, and the hyperechoic central scar is often evident. On unenhanced CT the lesion is of low attenuation, displaying heterogenous enhancement with intravenous contrast administration. The central scar is typically of low signal on both T1- and T2-weighted imaging, which can help differentiate it from FNH (whose scar typically is of low signal on T1 but high signal on T2-weighted imaging). The central scar is present in up to 60% of patients. Calcifications are present in up to 55% and are more common than in hepatocellular carcinoma.

55
Q
  1. A 53 year old male is investigated for recurrent episodes of biliary colic. Blood tests reveal eosinophilia and normal liver function tests. Abdominal ultrasound demonstrates a 7 cm cystic structure with a thin hyperechoic wall and several smaller satellite cysts up to 2 cm adjacent to the lesion. Which one of the following diagnoses is most likely?

a. Hydatid cyst

b. Pyogenic abscess

c. Amoebic abscess

d. Schistosomiasis

e. Hepatocellular carcinoma

A
  1. a. Hydatid cyst

The most likely diagnosis is hydatid cyst disease. This condition is caused by infection of the liver with the parasite Echinococcus granulosus. Blood eosinophilia is present in up to 50% of patients. It is more common in the right lobe of the liver and is multiple in 20% of cases. Daughter cysts are typical. Percutaneous aspiration of the cyst is positive for hydatid disease in 70%.

56
Q
  1. A 52 year old male has an unenhanced CT KUB for left renal colic. No cause for the pain is discovered on the CT, however the liver is found to be of increased density relative to the spleen. Which one of the following would be most likely to explain this incidental finding?

a. Excess alcohol intake

b. Amiodarone use

c. Diabetes

d. Steroids

e. Past history of chemotherapy

A
  1. b. Amiodarone use

The normal liver is between 30 and 70HU on unenhanced CT, and should be 10–15HU lower than spleen density. On portal venous phase the liver will be approximately 25HU less than the spleen. Amiodarone contains iodine and can cause the liver to appear of increased density on CT. Other causes include cisplatin use, haemochromatosis, Wilson disease and glycogen storage diseases. The more common finding on CT is a liver of decreased density due to a fatty liver. This has many causes including alcohol use, steroids, chemotherapy, diabetes and nutritional causes.

57
Q
  1. A 42 year old woman undergoes a CT abdomen and pelvis for the investigation of right upper quadrant pain and deranged liver function tests. On early post-intravenous contrast images there is prominent enhancement of the central liver and weak enhancement of the peripheral liver. This pattern is reversed on delayed images. In addition there is hypertrophy of caudate lobe. Which one of following would most likely explain these findings?

a. Acute hepatitis

b. Cirrhosis

c. Budd–Chiari syndrome

d. Portal hypertension

e. Fatty liver

A
  1. c. Budd–Chiari syndrome

Budd–Chiari syndrome is outflow obstruction of the hepatic veins due to a wide variety of causes, but two-thirds are idiopathic. CT features include ‘flip-flop’ enhancement pattern as described in the question, ascites, hepatosplenomegaly, gallbladder wall thickening and increased portal vein diameter. An enlarged caudate lobe is seen in up to 88%, which enhances normally due its venous drainage passing directly into the IVC.

58
Q
  1. A 61 year old man undergoes CT abdomen and pelvis for characterisation of a well-defined hyperechoic area seen on ultrasound in the perihilar region of the liver. On CT the area is of decreased attenuation but has no obvious mass effect. There is no abnormal enhancement with intravenous contrast administration. Which one of the following diagnoses is most likely?

a. Focal nodular hyperplasia

b. Focal fatty infiltration

c. Hepatic cyst

d. Liver haemangioma

e. Fibrolamellar carcinoma

A
  1. b. Focal fatty infiltration

Focal fatty infiltration occurs typically in the periportal and centrilobar regions of the liver and is commonest adjacent to the falciform ligament. Ultrasound features include a hyperechoic area with geographic margins. CT shows an area of decreased attenuation which does not alter the course of blood vessels or liver contour. The lesions are of high signal on T1-weighted MR imaging, and isointense or low signal on T2-weighted imaging. Haemangiomas would also typically be of increased echogenicity on ultrasound, but would be expected to show increased peripheral enhancement with intravenous contrast on CT.

59
Q
  1. A 41 year old female has an MRI liver following a solitary 3 cm lesion in the right lobe of the liver. The lesion is isointense on T1-weighted and slightly hyperintense to liver parenchyma on T2-weighted imaging. There is immediate intense homogenous enhancement with gadolinium in the arterial phase, which becomes isointense on the venous phase. A central scar is hypointense on T1 and hyperintense on T2-weighted sequences. Which one of the following is the most likely diagnosis?

a. Adenoma

b. Cavernous haemangioma

c. Fibrolamellar carcinoma

d. Regenerative nodules

e. Focal nodular hyperplasia

A
  1. e. Focal nodular hyperplasia

These imaging features are typical of focal nodular hyperplasia. This is the second most common benign liver tumour and typically occurs in women more often than in men. Adenomas are usually larger, enhance less brightly and do not typically have a central fibrous scar. Cavernous haemangiomas are usually high signal on T2-weighted images, and of blood pool intensity on contrast-enhanced T1-weighted images. Fibrolamellar carcinoma also has a central scar, but this is typically of low signal intensity on T2-weighted imaging. Regenerative nodules show high signal intensity on unenhanced T1-weighted imaging and do not have a scar.

60
Q
  1. A 71 year old woman with no significant past medical history has an abdominal ultrasound as part of an investigation for right upper quadrant pain, anaemia and weight loss. Multiple, poorly defined, markedly echogenic lesions are seen throughout the liver. Biopsy reveals these to be metastases. Which one of the following is most likely to be the primary tumour?

a. Adenocarcinoma of the colon

b. Melanoma

c. Invasive ductal carcinoma of the breast

d. Gastric cancer

e. Pancreatic ductal adenocarcinoma

A
  1. a. Adenocarcinoma of the colon

The most common primary tumours that cause brightly echogenic liver metastases are colonic adenocarcinoma, treated breast cancer and hepatoma. The differential here therefore lies between breast cancer and colon cancer. Colon cancer makes up at least 50% of highly echogenic metastases. In addition the question states that the patient has no significant past medical history, and therefore treated breast cancer is unlikely.?

61
Q
  1. A 25 year old female becomes unwell six hours after induced delivery for pre-eclampsia with severe right upper quadrant pain, oedema and nausea. CT of the abdomen and pelvis reveals copious ascites and multiple wedge-shaped areas of liver non-enhancement consistent with hepatic infarction. Which of the following is the most likely underlying cause?

a. Hepatic artery embolus

b. Portal vein thrombosis

c. SVC occlusion

d. HELLP syndrome

e. Splenic vein thrombosis

A
  1. d. HELLP syndrome

Hepatic infarction is rare because of the dual blood supply to the liver via the hepatic arterial system and the portal venous system. Isolated pathology in either of these vascular supplies is unlikely to cause hepatic necrosis as the other supply will usually compensate. HELLP is characterised by haemolysis, elevated liver enzymes and low platelets and is one of the causes of liver infarction.

62
Q
  1. A 47 year old female with a history of surgery for breast carcinoma is referred for ultrasound after liver function tests show a mildly elevated alkaline phosphatase. The bile ducts are normal but a 3 cm hyperechoic liver lesion is seen in the right lobe. CT is recommended, which shows a focal mass with nodular hyperenhancement of the periphery on arterial phase imaging becoming isointense to the background liver on delayed phase scanning at five minutes. Which one of the following is the most likely diagnosis?

a. Fibrolamellar carcinoma

b. Adenoma

c. Cavernous haemangioma

d. Adenocarcinoma metastases

e. Focal nodular hyperplasia

A
  1. c. Cavernous haemangioma

Metastases may show peripheral enhancement with complete fill-in on delayed images, but they typically show complete rather than nodular peripheral enhancement and washout on delayed phase imaging. Only haemangiomas typically show peripheral nodular enhancement. Cavernous haemangiomas are the most common benign liver tumours and are usually less than 4 cm in size. Seventy per cent are hyperechoic on ultrasound and they may show acoustic enhancement.

63
Q
  1. A 33 year old female presents to A&E with right upper quadrant pain, hypotensive and tachycardic. CT abdomen and pelvis reveals an 11 cm diameter well-defined heterogenous mass within the right lobe of the liver, predominantly of low density but with three focal areas of higher attenuation (>90 HU) within it. There is layered high attenuation fluid within the subhepatic and right subdiaphragmatic space tracking down to the pelvis. Which one of the following is the correct combination of recommendations?

a. Adenoma – recommend surgical referral

b. Adenoma – recommend correct coagulopathy and rescan if it deteriorates

c. Adenoma – recommend endovascular embolisation

d. Metastatic hepatocellular carcinoma – recommend gastroenterology referral

e. Trauma – needs CT thorax to clear other injuries

A
  1. c. Adenoma – recommend endovascular embolisation

Adenomas are vascular lesions comprising hepatocytes. They may occasionally present with massive haemorrhage, and are the most common liver lesion to do so in young people. In this scenario there is active extravasation of contrast implying active bleeding and haemoperitoneum. Urgent embolisation is the most appropriate treatment to halt bleeding. Conservative or surgical management is unlikely to provide rapid haemostasis. As a proportion of adenomas become malignant, they are usually removed surgically.

64
Q
  1. A 54 year old male has a liver MR for characterisation of a 3 cm low-attenuation lesion found on staging CT for rectal carcinoma. Which one of the following characteristics would be most worrying for a metastasis rather than a benign lesion?

a. Peripheral washout on delayed imaging

b. Intense arterial enhancement

c. Peripheral nodular enhancement

d. Presence of a pseudocapsule

e. Low signal intensity on T1-weighted imaging

A
  1. a. Peripheral washout on delayed imaging

Peripheral washout of contrast on delayed imaging is virtually diagnostic of malignancy. On post-gadolinium-enhanced T1-weighted images most metastases are hypovascular compared with the surrounding liver and are most conspicuous at the portal phase of enhancement. However, virtually all metastases exhibit a complete ring of peripheral enhancement, which is best seen in the early arterial phase.

65
Q
  1. A 63 year old male has a CT abdomen and pelvis for the investigation of change in bowel habit and weight loss. A sigmoid tumour is demonstrated and there is a solitary liver metastasis. Which one of the following observations on CT would render the patient ineligible for curative resection of the liver metastasis?

a. Presence of a single peripheral left lower lobe pulmonary metastasis

b. Presence of splenic metastasis

c. Direct extension of the liver metastasis into the right adrenal gland

d. Involvement of the caudate lobe

e. Peritoneal metastases

A
  1. e. Peritoneal metastases

Generally accepted contraindications to liver resection would include uncontrollable extrahepatic disease such as: non-treatable primary tumour; widespread pulmonary disease; locoregional recurrence; peritoneal disease; extensive nodal disease, such as retroperitoneal, mediastinal or portal nodes; and bone or CNS metastases. Patients with extrahepatic disease that should be considered for liver resection include: resectable/ablatable pulmonary metastases; resectable/a blatable isolated extrahepatic sites – for example, spleen, adrenal or resectable local recurrence; and local direct extension of liver metastases to, for example, the diaphragm or adrenal glands, which can be resected.

66
Q

A 54-ycar-old man with hepatitis B cirrhosis attends the hepatology outpatient clinic. The patient’s serum alpha fetoprotein level is found to be significantly elevated, having been normal 6 months ago. An abdominal ultrasound demonstrates a new 3-cm lesion in the right lobe of liver and a diagnosis of hepatocellular carcinoma (HCC) is suspected. Which one of the following statements is correct regarding HCC?

A Brain metastases are hypovascular and calcified.

B HCC derives its blood supply primarily from the hepatic artery.

C Portal vein invasion is more suggestive of a liver metastasis than HCC.

D Small HCC (< 1 cm) are typically heterogeneous and hyperechoic on US.

E The bony skeleton is the most common site for distant metastases.

A

E The bony skeleton is the most common site for distant metastases.

A localised paralytic ileus due to inflammatory change in the pancreas is known as the sentinel loop sign.

67
Q

A 54-ycar-old man with hepatitis B cirrhosis attends the hepatology outpatient clinic. The patient’s serum alpha fetoprotein level is found to be significantly elevated, having been normal 6 months ago. An abdominal ultrasound demonstrates a new 3-cm lesion in the right lobe of liver and a diagnosis of hepatocellular carcinoma (HCC) is suspected. Which one of the following statements is correct regarding HCC?

A Brain metastases ate hypovascular and calcified.

B HCC derives its blood supply primarily from the hepatic artery.

C Portal vein invasion is more suggestive of a liver metastasis than HCC.

D Small HCC (< 1 cm) are typically heterogeneous and hyperechoic on US.

E The bony skeleton is the most common site for distant metastases.

A

B HCC derives its blood supply primarily from the hepatic artery.

HCC derives its blood supply from the hepatic artery (hence the rapid arterial phase enhancement). A large HCC will usually demonstrate heterogeneous reflectivity due to areas of necrosis, but smaller lesions are typically of homogeneous low reflectivity on ultrasound. HCC often invades the branches of the portal vein and the most frequent site of metastases is the lungs. Metastases to the brain are typically hypervascular and do not usually contain calcification.

68
Q

A 53-year-old man has a history of type 2 diabetes mellitus and non-alcoholic steatohepatitis. He complains of weight loss and malaise; therefore anabdominal ultrasound is performed. This demonstrates a 2-cm focal lesioning the liver parenchyma. Which additional findings would be most consistent with an area of focal fatty sparing in hepatic steatosis?

A There is avid enhancement of the lesion on contrast-enhanced ultrasound.

B The focal area is of increased echogenicity, compared to that of the surrounding liver.

C The lesion has a geographic margin and is of reduced echogenicity compared to that of the surrounding liver.

D The lesion is hypoechoic with vessels displaced around its margins.

E The lesion is in the gallbladder fossa and is associated with segmental intrahepatic biliary dilatation.

A

C The lesion has a geographic margin and is of reduced echogenicity compared to that of the surrounding liver.

Focal fatty sparing usually leads to geographic or wedge-shaped lesions of reduced echogenicity. There should be no mass effect or abnormal enhancement characteristics in focal fatty sparing or infiltration. Both focal fatty sparing and infiltration typically occur in sites such as the gallbladder fossa, falciform ligament and porta hepatis, due to the altered venous drainage of these areas.

69
Q

A 68-year-old woman presents with malaise and abdominal pain and is found to have abnormal liver function tests. An abdominal ultrasound identifies multiple hyperechoic lesions in the liver and contrast-enhanced CT of the abdomen demonstrates that these are Hypervascular liver metastases. Given the CT findings, what is most likely diagnosis?

A Carcinoid

B Lymphoma

C Non-small-cell lung cancer

D Ovarian epithelial carcinoma

E Transitional cell carcinoma (TCC) of bladder

A

A Carcinoid

Most liver metastases are hypovascular and appear as lower attenuation than normal liver parenchyma on portal venous phase CT. Hypervascular metastases are less common and appear most prominently during the arterial phase (20-30 s) post contrast. Carcinoid, melanoma, thyroid and renal cancer are the most common causes of hypervascular liver metastases.

70
Q

A 35-year-old pregnant woman (28 weeks gestation) presents to her GP with right upper quadrant abdominal pain and is found to have abnormal liver function tests. An abdominal ultrasound is performed and demonstrates a diffusely hyperechoic liver with a discrete 4-cm hypoechoic lesion in the right lobe. An MRI is performed 3 weeks later, showing that the liver lesion has increased in size, now measuring 7 cm diameter. The lesion is isointense on in-phase Tlw images, losing signal on out-of-phase images. Following intravenous gadolinium there is immediate and intense enhancement with early washout. What is the most likely diagnosis?

A Choriocarcinoma

B Focal nodular hyperplasia

C Hepatic adenoma

D Liver haemangioma

E Metastatic breast cancer

A

C Hepatic adenoma

The MRI findings are highly suggestive of a hepatic adenoma and the history of rapid growth during pregnancy is supportive of this diagnosis.

71
Q

A 59-year-old man undergoes surgical resection of a rectal tumour. A contrast-enhanced CT of the abdomen is performed 3 months later and demonstrates a new, solitary 3-cm liver metastasis. The lesion lies inferior to the level of the left and right portal veins and posterior to the right hepatic vein. The remainder of the CT examination is unremarkable and the patient is assessed for surgical resection of the liver lesion. Which segment of the liver does the liver metastasis lie in?

A Segment 4b

B Segment 5

C Segment 6

D Segment 7

E Segment 8

A

C Segment 6

The Couinaud classification divides the liver into 8 independent segments, each of which has its own vascular supply and biliary drainage. The portal vein separates the superior and inferior segments, with further division based on the relationship to the nearest hepatic veins. Because of this division into self-containment’s, the Couinaud classification carries particular importance in the setting of resectable liver lesions, such as a solitary colorectal metastasis.

72
Q

A 38-year-old woman receives an orthotopic liver transplant for chronic liver failure due to primary biliary cirrhosis. The patient’s liver enzyme levels become markedly elevated after 24 hours and her clinical condition deteriorates. An abdominal ultrasound is performed with Doppler evaluation of the hepatic vessels. Given the clinical history, which vascular complication is most likely to have occurred?

A Arterioportal fistula

B I VC thrombosis

C Hepatic artery stenosis

D Hepatic artery thrombosis

E Portal vein thrombosis

A

D Hepatic artery thrombosis

Hepatic artery thrombosis is the most common and serious early vascular complication post liver transplant. Thrombolysis is an option in this setting, but many patients will ultimately require transplantation. Prompt and accurate diagnosis is therefore essential. A parvus et tardus waveform may be seen distal to stenosis of the hepatic artery, with associated elevated systolic velocity at the stenotic segment. Porta! vein thrombosis is less common and can result in the acute development of ascites and varices. IVC thrombosis is rare and usually related to surgical technique, whilst an arterioportal fistula is an infrequent complication of liver biopsy.

73
Q

A 37-year-old in an presents to his GP with increasing right upper quadrant pain. On examination, he is afebrile with right upper quadrant tenderness and fullness. An abdominal ultrasound is performed and demonstrates a5-cm diameter cystic lesion in the right lobe of liver. The mass contains multiple septations with a large cyst centrally and multiple small cystic spaces peripherally. Echogenic debris is seen within the cystic lesion and alters in position when the patient lies on his side. From the clinical and sonographic details, what is the most likely diagnosis?

A Amoebic abscess

B Hydatid cyst

C Pyogenic liver abscess

D Simple liver cyst

E Solitary metastasis

A

B Hydatid cyst

A multiloculated cystic mass with daughter cysts and echogenic debris (‘hydatid sand’) is characteristic of a hydatid liver cyst.

74
Q

A 78-year-old man has myelodysplastic syndrome and requires frequent blood transfusions. He develops progressively abnormal liver function tests and a grossly elevated ferritin level. An MRI of the liver is performed using breath hold half Fourier single shot spin echo T2w images. Which finding would make a diagnosis of hemosiderosis (iron overload from recurrent blood transfusion) more likely than haemochromatosis?

A Increased T2 signal in the liver only

B Increased T2 signal in the liver and spleen

C Reduced T2 signal in the liver only

D Reduced T2 signal in the liver and spleen

E Reduced T2 signal in the spleen only

A

D Reduced T2 signal in the liver and spleen

In iron overload due to recurrent transfusions, there is increased iron deposition in the reticuloendothelial system. This leads to reduced Tl, T2 andT2* signal intensity in the liver and spleen. Haemochromatosis causes diffusely reduced T2 signal in the liver and may lead to cirrhosis, but the splenic signal intensity should remain normal. Diffuse fatty liver will lead to increased T2 signal in the liver with signal loss during out-of-phase images.

75
Q

A 22-year-old woman presents to her GP with a 4-month history of increasing right upper quadrant pain. An abdominal ultrasound is performed and demonstrates a 6-cm solid lesion of increased reflectivity in segment 6 of the liver. A contrast-enhanced CT of the liver is performed and demonstrates that the lesion enhances moderately and has a lobulated margin. Which additional finding would make a diagnosis of fibrolamellar carcinoma more likely than that of focal nodular hyperplasia (FNH)?

A A hyperechoic central scar

B A preexisting history of chronic liver disease

C Delayed enhancement of a central scar

D Punctuate calcification in the lesion

E The patient is talking the combined oral contraceptive pill

A

D Punctuate calcification in the lesion

There is considerable overlap in the imaging appearances of these two conditions, but punctate calcification occurs in over half of patients with fibrolamellar carcinoma and is extremely unusual in FNH.

76
Q

A 42-year-old man is admitted to hospital with acute abdominal pain. There is a significant medical history of polycythemia rubravera, for which the patient undergoes regular venesection. On examination, there is right upper quadrant tenderness and hepatomegaly. Liver function tests are acutely elevated and the patient’s condition deteriorates. A catheter angiogram is performed to assess the major hepatic vessels and shows a ‘spider’s web ‘appearance within the liver. What is the diagnosis?

A Budd Chiari syndrome

B Capillary haemangioma

C Hereditary hemorrhagic telangiectasia (HHT)

D Portal vein thrombosis

E Spontaneous hepatic hematoma

A

A Budd Chiari syndrome

When contrast is injected into the hepatic veins, a ‘spider’s web’ appearance of collateral vessels is diagnostic of Budd Chiari syndrome

77
Q

A 74-year-old woman is referred to the hepatology outpatient clinic with persistently abnormal liver function tests. There is a past medical history of myocardial infarction, atrial fibrillation and hypertension, but no previous history of liver disease. On abdominal ultrasound, the liver appears normal with antegrade portal venous flow demonstrated. A CT of the abdomen is performed and the mean density of the liver is 86 Hounsfield Units (HU) precontrast. What is the most likely diagnosis?

A Amiodarone therapy

B Chronic Budd Chiari syndrome

C Chronic hepatitis B

D Diffuse fatty infiltration

E Wilson’s disease

A

A Amiodarone therapy

Amiodarone contains iodine; therefore, deposition in the liver leads to increased density on CT.

78
Q

A 31-year-old woman has a 6-month history of intermittent right upper quadrant pain. An abdominal ultrasound examination is performed and reveals a 3-cm hyperechoic mass in segment 6 of the liver. She undergoes an MRI examination of the liver with intravenous gadolinium. On the precontrast Tlw images, the signal intensity of the lesion is isointense to surrounding liver parenchyma. Which one of the following statements is true regarding the post-gadolinium Tlw images?

A Focal fatty infiltration demonstrates enhancement in the arterial phase.

B HCC usually enhance in the delayed phase images only.

C Hepatic adenomas typically demonstrate uniform enhancement in the arterial phase.

D Hypervascular metastases are typically hyperintense on T1 precontrast.

E Progressive centripetal enhancement in the portal venous and delayed phases is seen with HCC.

A

C Hepatic adenomas typically demonstrate uniform enhancement in the arterial phase.

Hepatic adenoma, HCC and focal nodular hyperplasia will usually show marked enhancement in the arterial phase. Hypervascular liver metastases are typically hypointense on Tl, hyperintense on T2. The characteristic behavior of a liver haemangioma—not HCC—is described in option E.

79
Q

A 53-ycar-old man is seen in the liver transplant outpatient clinic. Two years ago, he underwent an orthotopic liver transplant for alcoholic liver disease and currently takes oral cyclosporin. He reports a 3-month history of weight loss and his liver function tests are found to be abnormal. A contrast-enhanced CT demonstrates multiple new low attenuation lesions within the liver. There is also marked thickening of several small bowel loops. What is the most likely diagnosis?

A Chronic graft ischaemia with portal hypertension

B Cyclosporin hepatotoxicity

C Multifocal hepatocellular carcinoma

D Post transplant lymphoproliferative disorder (PTLD)

E Secondary amyloidosis

A

D Post transplant lymphoproliferative disorder (PTLD)

Up to 5% of liver transplants develop PTLD: extra nodal disease is the most common pattern.

80
Q

A 27-year-old man is referred to the hepatology outpatient clinic with a 3-weekhistory of malaise, lethargy and mild upper abdominal pain. Liver function tests performed by his GP are significantly abnormal. The results of hepatitis serology performed in the clinic are consistent with an acute hepatitis B infection. An abdominal ultrasound is performed. What is the most likely finding on ultrasound?

A Decreased reflectivity of the liver parenchyma

B Increased reflectivity of the liver parenchyma

C Nodular liver surface

D Normal ultrasound appearances

E Retrograde portal venous flow

A

D Normal ultrasound appearances

In acute viral hepatitis, there can be diffusely reduced reflectivity of the liver but the majority of patients have a normal ultrasound examination.

81
Q

A 74-year-old woman is referred to hospital by her GP as an emergency medical admission. The referral letter indicates that the patient is in residential care and has Alzheimer’s disease. Her carers have noticed generalized malaise and significant weight loss over the past 6 weeks. A contrast-enhanced CT is performed and demonstrates multiple low attenuation liver metastases. These lesions contain foci of amorphous calcification and show rim enhancement in the portal venous phase.
What is the most likely underlying malignancy in this patient?

A Carcinoid tumour of ileum

B Endometrial carcinoma

C Mucinous adenocarcinoma of colon

D Multifocal hepatocellular carcinoma

E Papillary carcinoma of thyroid

A

C Mucinous adenocarcinoma of colon

Calcification occurs in 2—3% of liver metastases. Mucinous adenocarcinoma of the GI tract is the most frequent underlying primary tumour.

82
Q

A 46-year-old woman was diagnosed with breast cancer 3 months ago. A recent abdominal ultrasound identified a solitary liver lesion and an MRI of the liver is performed. This demonstrates a 2.5-cm diameter mass in liver segment 8. This lesion has a well-defined, lobulated contour and yields highT2 signal. An extended echo time of 180 ms is used and the lesion remains of high T2 signal (greater than the spleen, less than CSF). What is the most
likely diagnosis?

A Breast cancer metastasis

B Focal nodular hyperplasia

C Hemangioma

D Hepatic adenoma

E Simple liver cyst

A

C Hemangioma

Extended echo times will emphasize the high T2 signal intensity of liver hemangiomas, in comparison with surrounding structures.

83
Q

A 59-year-old woman is found to have several small lesions within the liver on abdominal ultrasound. The ultrasound had been requested for investigation of abnormal liver function tests. An MRI of the liver demonstrates several low T2 signal lesions within the liver parenchyma. These lesions yield high signal on Tlw images and, following intravenous gadolinium, there is avid enhancement in the hepatic arterial phase. Which diagnosis would best explain these findings?

A Colorectal metastasis

B Focal fatty infiltration

C Focal fatty sparing

D Melanoma metastases

E Multifocal HCC

A

D Melanoma metastases

Melanoma metastases have a different appearance to most liver metastases as the paramagnetic effect of melanin leads to high Tl and low T2 signal.

84
Q
  1. A lesion is noted in the liver on CT and ultrasound. It is inferior, anterior, and to the left of the right hepatic vein, but to the right of the middle hepatic vein. It is inferior of the confluence of the right and left portal veins. According to the Couinaud system, what segment of the liver is the lesion in?

A. Segment 4b.

B. Segment 5.

C. Segment 6.

D. Segment 7.

E. Segment 8.

A
  1. B. Segment 5.

For a review of the segmental anatomy of the liver, please see the reference below.

85
Q
  1. A 56-year-old male patient is referred for an ultrasound of abdomen prior to undergoing an anterior resection for a proximal rectal carcinoma. The ultrasound reveals a 2cm lesion in the right lobe of the liver, which is hyperechoic centrally with a hypoechoic rim. Which one of the following cannot be considered in the differential for this lesion?

A. Metastases.

B. Haemangioma.

C. Sarcoid.

D. Candidiasis.

E. Lymphoma.

A
  1. B. Haemangioma.

The ultrasound findings describe a target lesion or bull’s eye lesion. Cavernous haemangioma scan have unusual appearances, but a small lesion such as described will normally have a uniform hyperechoic appearance. Of all the other lesions described, metastasis would be top of the differential, although this appearance is not the most classical for a colonic metastasis.

86
Q
  1. A 40-year-old female undergoes MRI of the liver, which demonstrates a5-cm lesion that is isointense to liver on T1WI and slightly hyperintense onT2WI. It has a central scar that is hypointense on T1WI and hyperintense on T2WI. On contrast-enhanced dynamic MRI, the lesion is hyperintense in the arterial phase, and isointense to liver in the portal venous phase with delayed filling in of the central scar. What is the diagnosis?

A. Hepatic adenoma.

B. Fibrolamellar hepatoma.

C. Hypervascular metastasis.

D. Focal nodular hyperplasia (FNH).

E. Giant haemangioma.

A
  1. D. Focal nodular hyperplasia (FNH).

This is the second most common benign liver tumour. It is thought to represent a hyperplastic response of hepatocytes to an underlying vascular malformation. It is most common in young adult females and is usually an asymptomatic solitary lesion. On histology, FNH consists of hyperplastic hepatocytes and small bile ductulus around a central scar. The bile ductulus of FNH do not communicate with the adjacent biliary tree. At ultrasound, FNH is isoechoic or hypoechoic. Colour Doppler may show prominent central vascularity. At CT, FNH is typically slightly hyperattenuating or isoattenuation to surrounding liver on precontract images. On postcontrast images, FNH is hyperattenuating in the arterial phase and isoattenuation in the portal venous phase with hypoattenuating central scar. The scar shows delayed enhancement. At MRI, FNH is iso- to hypointense on T1WI and slightly hyper- to isointense on T2WI. The central scar is hypointense on T1WI and hyperintense on T2WI. The enhancement pattern is similar to that on CT. If the appearances are atypical, MRI with hepatocyte-specific contrast agent (gadobenatedimeglumine) may be useful in confirming the hepatocellular origin of the mass. With gadobenatedimeglumine, FNH is iso- to hyperintense on the 1–3 hour delayed images in over 96% of cases.

87
Q
  1. A 45-year-old female is suspected to have focal areas of fat infiltration on ultrasound of the liver. An MRI of the liver is requested for further assessment. What sequences are most useful in confirming the diagnosis of focal fat infiltration?

A. T1WI pre and post gadolinium.

B. T1WI and T2WI.

C. T1WI and fat-saturated T2WI.

D. Dual GE T1WI in phase and out of phase.

E. MR spectroscopy.

A
  1. D. Dual GE T1WI in phase and out of phase.

Three basic MRI techniques are available for fat detection, which work on the basis of the difference in precessional frequency between water and fat protons. These are chemical shift imaging, frequency-selective imaging, and MR spectroscopy. Dual GE T1WI is the most useful sequence in clinical practice. It is based on the phase interference effect or chemical shift imaging of the second kind. When the fat and water protons are in phase there is constructive interference and when they are out of phase there is destructive interference. By comparing the signal intensities on the in-phase and out-of-phase images, fat detection is possible. Chemical shift imaging of the first kind, or chemical shift spatial misregistration, occurs at fat–water interfaces in the frequency-encoding direction, manifesting as alternating bands of high and low signal. It is present in all standard non-fat-saturated sequences, but it can be subtle and may be missed or mistaken for image noise. Frequency-selective imaging with selective excitation or saturation depends on the homogeneity of the magnetic field and the size of the lesion.MR spectroscopy is too time-consuming for routine clinical use.

88
Q
  1. A 55-year-old female with cirrhosis undergoes MRI of the liver, which demonstrates multiple small nodules that are hypointense on T2WI and enhance following administration of gadolinium in the arterial and portal venous phase. The nodules demonstrate uptake of hepatocellular agent and super paramagnetic iron oxide (SPIO) particles. What is your diagnosis?

A. Multifocal hepatocellular carcinoma (HCC).

B. Siderotic nodules.

C. Dysplastic nodules.

D. Regenerative nodules.

E. Multiple arterio-venous shunts.

A
  1. D. Regenerative nodules.

Regenerative nodules are formed in response to necrosis and altered circulation. They remain enhanced in the portal venous phase as opposed to HCC, which typically demonstrates contrast washout in the portal venous phase. Regenerative nodules have normal hepatocellular function and Kupffer cell density and therefore demonstrate uptake of both hepatocellular agents and SPIO particles. As dedifferentiation proceeds, the hepatocellular function and Kupffer cell density reduce.

89
Q
  1. A 62-year-old male with acute myocardial infarction develops abdominal discomfort and deranged liver function tests. A CT scan of the abdomen demonstrates heterogeneous liver enhancement, poor enhancement of the hepatic veins and inferior vena cava (IVC), ascites and binasal pleural effusions. What additional feature would favor a diagnosis of passive hepatic congestion instead of acute Budd–Chiari syndrome?

A. Flip-flop enhancement pattern of the liver.

B. Absent flow in hepatic veins.

C. Dilated hepatic veins and IVC.

D. Enlarged caudate lobe.

E. Hepatomegaly.

A
  1. C. Dilated hepatic veins and IVC.

Elevated right atrial/central venous pressure due to cardiac decompensation results in impaired venous drainage from the liver, producing passive hepatic congestion. If prolonged, passive hepatic congestion can result in cardiac cirrhosis. On CT imaging, retrograde enhancement of dilated IVC and hepatic veins is seen in the arterial phase. In the portal venous phase, there is delayed/reduced enhancement of the hepatic veins due to impaired venous drainage. There is heterogenous enhancement of the liver parenchyma due to venous stasis. Other features of cardiac failure may be evident. Acute Budd–Chiari syndrome is characterized by narrowed hepatic veins and intrahepatic IVC (secondary to compression by the enlarged liver) and by flip-flop pattern of enhancement between the arterial and the portal venous phases.

90
Q
  1. A 47-year-old male patient undergoes an MRI examination for further characterization of an adrenal lesion. Axial gradient T1 in- and out-of-phase sequences confirm the benign nature of the adrenal lesion. Incidentally, the liver and pancreas demonstrate a signal drop on the in-phase images compared to out-of-phase images. What is your diagnosis and what additional sequence would confirm the diagnosis?

A. Diffuse fatty infiltration. GE T2WI.

B. Diffuse fatty infiltration. SE T2WI.

C. Haemochromatosis. SE T2WI.

D. Haemochromatosis. GE T2WI.

E. Hemosiderosis. GE T2WI.

F. Hemosiderosis. SE T2WI.

A
  1. D. Haemochromatosis. GE T2WI.

A dual GE T1 in- and out-of-phase sequence is routinely used in identifying lipid content within an adrenal lesion. It is based on the phase interference effect. When the fat and water signals are in-phase, there is constructive interference and when they are out-of-phase there is destructive interference. This results in signal drop-off on the out-of-phase sequence. The reverse effect of decreased signal intensity on in-phase images compared to out-optacons is seen in iron deposition diseases. This is because the echo time for the in-phase sequence is longer than for the out-of-phase sequence, therefore the in-phase sequence is more susceptible to the paramagnetic (dephasing) effects of iron. Haemochromatosis is an autosomal recessive genetic disorder. There is abnormal deposition of iron in parenchymal organs such as the liver, pancreas, heart, etc. In hemosiderosis or secondary haemochromatosis, iron deposition is seen in the reticuloendothelial system of the liver, spleen, and bone marrow. This type of deposition is not associated with tissue damage. A GE T2 sequence demonstrates signal loss due to the magnetic field inhomogeneity produced by the paramagnetic effects of iron. GE sequences are more susceptible to paramagnetic effects than SE sequences, as there is no 180°rephasing pulse in gradient sequences.

91
Q
  1. A 50-year-old male undergoes MRI of the liver for further characterization of a suspected haemangioma on ultrasound. In addition to the haemangioma, a peripheral wedge-shaped area of enhancement is seen in the arterial phase but no abnormality is seen in the corresponding area in the non-contrast or portal venous phases. What is the diagnosis?

A. Hepatocellular carcinoma.

B. Hepatic infarct.

C. Transient hepatic intensity difference (THID).

D. Hypervascular metastasis.

E. Haemangioma.

A
  1. C. Transient hepatic intensity difference (THID).

THID on MRI or transient hepatic attenuation difference (THAD) on CT is a pseudo lesion caused by focal alteration in the hemodynamics of the liver due to either non-tumorous arterio-portal shunt or obstruction of distal portal venous flow. THID or THAD is seen as a focal area of enhancement in the arterial phase only, with no abnormality seen in the portal venous phase. Features suggestive of THID or THAD include peripheral location, wedge shape, straight margins, and normal vessels coursing through the area.

92
Q
  1. A 50-year-old male patient is admitted with congestive cardiac failure and undergoes a CT scan of the abdomen, which shows tortuous and prominent intrahepatic and extrahepatic arterial branches with early filling of dilated hepatic veins and IVC. The arterial phase scan shows mosaic perfusion with multiple enhancing foci. In the portal venous phase there is homogenous enhancement of the liver, with the prominent hepatic veins and IVC noted. What is the diagnosis?

A. Passive hepatic congestion.

B. Budd–Chiari syndrome.

C. Osler–Weber–Rendu syndrome.

D. Multifocal transient hepatic attenuation differences.

E. Von Meyer burg complex.

A
  1. C. Osler–Weber–Rendu syndrome.

Osler–Weber–Rendu syndrome or hereditary hemorrhagic telangiectasia (HHT) is a rare autosomal dominant multisystem vascular disorder characterized by angiodysplastic lesions in which there is communication between arteries and veins of varying sizes. It commonly affects the skin, lungs, and mucous membranes but any organ system may be involved. The liver is the most common site of abdominal HHT. Lesions range from tiny telangiectases to transient perfusion abnormalities and large confluent vascular masses. Coronal maximum intensity projection (MIP) images are useful in appreciating telangiectases. Liver involvement is associated with arterio-venous shunting, porto-venous shunting, or both, resulting in hyperdynamic circulation, which may lead to high-output cardiac failure. Budd–Chiari syndrome is hepatic vein thrombosis and Von Meyer burg complex is multiple biliary hamartomas.

93
Q
  1. A 55-year-old male liver transplant recipient undergoes Doppler ultrasound assessment at 1 year for deranged liver function tests. Colour Doppler imaging demonstrates a stenosis in the hepatic artery, at the presumed anastomosis. Which of the following statements with regard to the associated pulsed Doppler findings is likely to be false?

A. Tardus-parvus arterial waveform distal to the stenosis.

B. Resistive index of 0.9 distal to the stenosis.

C. Spectral broadening in the immediate post-stenotic portion.

D. Elevated peak systolic velocity at the stenosis.

E. Elevated end diastolic velocity at the stenosis.

A
  1. B. Resistive index of 0.9 distal to the stenosis.

Knowledge of the vessel ‘waveform signature’ and stenosis flow dynamics is essential in interpreting the Doppler findings. The hepatic artery is a low resistance vessel with continuous antegrade flow during systole and diastole. Generally, low-resistance arteries have a resistive index (RI) of 0.55–0.7. RIs higher or lower than this range are abnormal. RI is calculated by using the formula RI = (peak systolic velocity – end diastolic velocity)/peak systolic velocity. Most modern ultrasound machines calculate this automatically. A stenosis results in increased arterial resistance to the blood flow proximal to the stenosis. This causes a reduction in the end diastolic velocity disproportionately more than the peak systolic velocity, producing a high-resistance waveform and high RI (due to a greater difference between the peak systolic and end diastolic velocities). This finding is not specific to stenosis and may be seen in the postprandial state, and in patients of advanced age and diffuse peripheral microvascular disease or compression (cirrhosis, hepatic venous congestion, cold ischaemia, and any stage of transplant rejection). At and immediately distal to the stenosis there is turbulent flow and a jet phenomenon resulting in an increase in the peak systolic and end diastolic velocity and spectral broadening. Depending on the severity of the stenosis, the artery distal to the stenosis demonstrates the following findings: Tardus-parvus waveform that refers to the late and low systolic peak (i.e. increased acceleration time and reduced peak systolic velocity). Low RI due to a greater reduction in peak systolic velocity compared to the end diastolic velocity. Low RI may also be seen with distal vascular shunts (trauma, iatrogenic, cirrhosis, Osler–Weber–Rendu syndrome).

94
Q

A young man was involved in a fight in a fast food outlet and was stabbed in the abdomen with a large knife. A CT of his chest, abdomen and pelvis showed a duodenal laceration with associated pneumoperitoneum and intraperitoneal free fluid. What another organ is most likely to have been injured?

a Colon

b Liver

C Pancreas

d Small bowel

e Spleen

A

Answer B: Liver

Duodenal injury is frequently associated with injury to other abdominal structures. The pattern of these associated injuries varies between blunt and penetrating trauma:
blunt trauma: pancreas > liver > spleen > colon > small bowel > kidney.
penetrating trauma: liver > pancreas > small bowel > kidney > colon > spleen.

95
Q

A diabetic 50-year-old man was investigated for hepatic disease. His liver was enlarged and he had hyperpigmentation of his skin. His serum iron was >300mg/dL and serum transferrin saturation were >50%. What imaging feature is most characteristic of the underlying diagnosis?

a Increase in hepatic echogenicity on US

b Patchy echogenicity demonstrated on US

C Decrease in hepatic Ti signal on MR

d Decrease in hepatic attenuation on CT

e Decrease in hepatic T2 signal on MR

A

Answer E: Decrease in hepatic T2 signal on MR

Haemochromatosis can be primary or secondary. The primary form shows autosomal recessive inheritance and usually presents after middle age, particularly in women where menstruation is protective. Hepatomegaly and hyperpigmentation are seen in 90%. Diabetes can occur in up to 30% secondary to pancreatic beta cell damage. Significant signal loss on T2WI is very characteristic.

96
Q

A front-seat passenger in a head-on collision was brought to the Emergency Department. A contrast-enhanced CT of their abdomen and pelvis showed a liver laceration in segment 6 that is 2 cm deep and 8 cm long. What grade is appropriate?

a Grade I laceration

b Grade II laceration

C Grade III laceration

d Grade IV laceration

e Grade V laceration

A

b Grade II laceration

97
Q

Following a holiday in South-East Asia, a 33-year-old man presented with abdominal pain. Ultrasound demonstrated an echo-poor lesion within the right lobe of the liver which contained several smaller internal cysts. What is the most likely diagnosis?

a Amoebic abscess

b Echinococcus

C Pyogenic abscess

d Polycystic liver disease

e Haemangioma

A

b Echinococcus

98
Q

A 67-year-old woman noticed an abdominal mass and was referred for an ultrasound scan. This showed a large, uniformly hyperechoic mass in the left lobe of the liver. A subsequent multi-phase contrast-enhanced CT confirmed this mass which enhances peripherally until it becomes isoattenuating with the adjacent liver parenchyma after five minutes. What is the most likely diagnosis?

a Adenoma

b Haemangioma

C Hepatocellular carcinoma

d Focal nodular hyperplasia

e Solitary metastasis

A

Answer B: Haemangioma

Hepatic haemangioma are the commonest benign liver tumours. They are typically hyperechoic on ultrasound and demonstrate centripetal enhancement with delayed `fill-in’ on contrast-enhanced CT. Enhancing focal liver lesions can be usefully characterised according to the phase of their enhancement. Other lesions that show delayed (equilibrium phase) enhancement include: cholangiocarcinoma, solitary fibrous tumour and treated metastases.

99
Q

A 38-year-old woman presented with upper abdominal discomfort. She was otherwise well and was taking the combined oral contraceptive pill. Physical examination was normal. An ultrasound showed a well-circumscribed, homogeneous 4-cm mass in the liver and she was subsequently referred for a liver MRI. What feature would be most useful in distinguishing focal nodular hyperplasia from adenoma?

a Calcification

b Internal haemorrhage

C Multiple lesions

d Uptake of reticuloendothelial contrast

e Uptake of hepatobiliary contrast

A

Answer E: Uptake of hepatobiliary contrast

Reticuloendothelial contrast agents, such as Ferrixan, carboxydextran-coated iron oxide nanoparticles (Resovist®) are taken up by both focal nodular hyperplasia (FNH) and adenoma. Hepatobiliary contrast agents, such as Gadolinium-Benzyloxypropionic tetra-acetate (Gd-BOPTA or MultiHance®) are not taken up by adenoma.

100
Q

A middle-aged man with symptoms of lethargy and painful hands presented to his doctor. Routine blood tests showed deranged liver function and plain radiographs of his hands showed an arthropathy primarily affecting the first and second metacarpophalangeal (MCP) joints. He was referred to a hepatologist and an abdominal CT was arranged. The liver was of diffusely increased density prior to contrast administration. What is the most likely underlying diagnosis?

a Amiodarone treatment

b Glycogen storage disorder

C Haemochromatosis

d Primary biliary cirrhosis

e Rheumatoid arthritis

A

Answer C: Haemochromatosis

Causes of increased liver attenuation pre-intravenous contrast include: haemochromatosis, haemosiderosis, iron overload, glycogen storage disease and amiodarone treatment.

101
Q

A 58-year-old African woman was visiting her grandchildren in London when she developed right upper quadrant pain. The local doctor, suspecting gallstones, arranged an ultrasound which showed a grossly cirrhotic liver with markedly hyperechoic septa separating areas of relatively normal liver. The scanning radiologist recognised this as a `turtle back’ appearance. What is the most likely underlying diagnosis?

a Amoebiasis

b Hydatid

C Lymphoma

d Schistosomiasis

e Tuberculosis

A

Answer D: Schistosomiasis

The described ultrasound appearances are typical of schistosomiasis. Radiological features rarely develop until the late stages and are signs of cirrhosis and fibrosis. Patients are at a significantly increases risk of hepatocellular carcinoma (HCC).

102
Q

A 17-year-old male developed upper abdominal pain and abdominal distension two days after an appendicectomy. He had been dehydrated on presentation but the operation was uncomplicated. Transabdominal ultrasound showed a large volume of ascites and a diagnosis of portal vein thrombosis was considered. What sonographic finding is most commonly seen that would support this diagnosis?

a Portosystemic collateral circulation

b Increase in portal vein diameter

C Echogenic material in the portal vein lumen

d Enlargement of a thrombosed portal vein above 15 mm

e Enlargement of the lesser omentum

A

Answer C: Echogenic material in the portal vein lumen

Echogenic material in the portal vein lumen (67%), increased in portal vein diameter (57%), portosystemic collateral circulation (48%), enlargement of a thrombosed portal vein above 15 mm (38 %).

103
Q

38 A 35-year-old male with protein C deficiency presented with a history of a few weeks of abdominal pain, increasing abdominal distension and jaundice. Biochemistry revealed grossly abnormal liver function tests and his coagulation was also markedly abnormal. An ultrasound demonstrated widespread ascites with hypertrophy of the caudate lobe and bicoloured flow in the hepatic veins on Doppler. What is the most likely diagnosis?

a Acute Budd-Chiari syndrome

b Alcoholic liver disease

c Chronic Budd-Chiari syndrome

d Portal vein thrombosis

e Right heart failure

A

38 Answer C: Chronic Budd-Chiari syndrome

In acute Budd-Chiari syndrome the caudate lobe does not hypertrophy as it does in the chronic form (as it has separate drainage directly into the IVC). There are various causes of Budd-Chiari syndrome; 66% are idiopathic.

104
Q

39 A young woman developed haematuria and was referred for a renal tract ultrasound. She was not on any medication other than the combined oral contraceptive pill. There were no positive renal tract findings, but while scanning the right kidney the ultrasonographer noticed a solitary isoechoic lesion in the right lobe of the liver. Following further assessment, the patient was referred for a contrast-enhanced MRI, primarily to discriminate between focal nodular hyperplasia (FNH) and adenoma. What radiological finding would support a diagnosis of FNH?

a Internal haemorrhage

b Calcification

C Central scar

d Poor uptake of hepatobiliary liver contrast (Gd-BOPTA)

e Uptake of reticuloendothelial liver contrast

A

39 Answer C: Central scar

105
Q

40 A 42-year-old man with known alcohol-related cirrhosis was assessed in the Hepatology outpatient clinic. Blood samples were taken for serum bilirubin, albumin and prothrombin time and evidence of hepatic encephalopathy was sought clinically. An abdominal ultrasound was requested to complete ChildPugh cirrhosis scoring. What feature of ultrasound is used to complete the Child-Pugh score?

a Ascitic fluid

b Collateral vessel formation

C Portal venous flow

d Splenomegaly

e Varices

A

40 Answer A: Ascitic fluid

The Child-Pugh score is derived from bilirubin and albumin levels, prothrombin time, hepatic encephalopathy and presence ascites.

106
Q

41 An adult patient presented with non-specific symptoms suggestive of malignancy and an abdominal ultrasound showed multiple `target-lesions’ throughout the liver consistent with hepatic metastatic deposits. Without knowing the patient’s age or gender, what is the most likely site of the primary tumour?

a Breast

b Colorectal

c Bronchogenic

d Pancreatic

e Gastric

A

41 Answer B: Colorectal

Colorectal > Gastric > Pancreatic > Breast > Lung.

107
Q

42 A gap-year student returned from a long trip to Africa having experienced a few episodes of bloody diarrhoea. She then developed increasingly severe right upper quadrant pain and was referred for an abdominal ultrasound. This showed a solitary 8-cm, hypoechoic thin-walled lesion in the right lobe of the liver. Subsequent percutaneous drainage yielded reddish brown thick fluid. What is the most likely causative organism?

a Echinococcus sp.

b Entannoeba histolytica

C Escherichia coli

d Schistosonna sp.

e Staphylococcus aureus

A

42 Answer B: Entarnoeba histolytica

Amoebic abscesses typically present as a large solitary lesion in the right lobe of the liver. The contents have been described as anchovy paste' or chocolate sauce’.

108
Q

43 A 28-year-old female complained of right upper quadrant pain, dyspnoea and abdominal distension shortly after a normal vaginal delivery of her first child. Transabdominal ultrasound showed ascites and an enlarged (14cm) spleen. What is the most likely diagnosis?

a Pulmonary embolus

b Cirrhosis

c Portal vein thrombosis

d Gastritis

e Budd-Chiari syndrome

A

43 Answer E: Budd-Chiari syndrome

Occlusion of the hepatic veins/IVC, often due to thrombosis in hypercoagulable states such as pregnancy. Hepatosplenomegaly, with caudate hypertrophy, are early sonographic signs with non visualisation of the hepatic veins in 75%. Symptoms also include right heart failure and pulmonary oligaemia.

109
Q

44 A 17-year-old Greek boy presented with abdominal pain. He was of short stature but otherwise of normal physical appearance. Blood tests showed a microcytic anaemia and abnormal liver function tests. On ultrasound his liver measured 18 cm in cranio-caudal length; in the right midclavicular line his spleen measured 15 cm in longest axis. Gallstones were also noted in an otherwise normal gallbladder. What is the most likely diagnosis?

a Sarcoidosis

b Glycogen storage disease

c Beta-thalassaemia

d Recent viral infection

e Leukaemia

A

44 Answer C: Beta-thalassaemia

All the options are causes of hepatosplenomegaly. Beta-thalassaemia is more common in the Mediterranean population and can cause microcytic anaemia and abnormal liver function. Affected patients are at increased risk of developing gallstones.

110
Q

2 A patient attending for an abdominal ultrasound scan was noted to have a focal abnormality within the liver, which lies high within the right lobe and abuts the right hepatic vein. Which segment is the lesion most likely to involve?

a 4A

b 4B

c 5

d 6

e 7

A

e 7

111
Q

37 A middle-aged female underwent an abdominal ultrasound which demonstrated an area of increased reflectivity within the right lobe of her liver. She has no significant past medical history and a benign haemangioma is suspected. Were she to have an MRI what would the most likely signal characteristics of this lesion be?

a HypointenseTi and hyperintense T2

b HypointenseTi and hypointense T2

c Hyperintense Ti and hyperintense T2

d Hyperintense Ti and hypointense T2

e Hyperintense Ti and isointense T2

A

a HypointenseTi and hyperintense T2

112
Q

39 Which of the following is an expected normal finding following liver transplant?

a Hepatic arterial resistive index <0.5

b Hepatic infarction

c Increased periportal attenuation

d Periportal lymphadenopathy

e Portal vein thrombosis

A

39 Answer C: Increased periportal attenuation

113
Q

40 A focal liver abnormality was detected in a middle-aged man and he subsequently underwent a liver MRI to further delineate the lesion. The MRI showed the lesion to be hyperintense on Ti - weighted sequences and hypointense on T2-weighted sequences. What lesion is most likely to exhibit this pattern of signal return?

a Adenoma

b Regenerative nodule

c Focal nodular hyperplasia

d Haemangioma

e Metastases

A

40 Answer A: Adenoma

Most focal liver lesions are hypointense on T1WI. Exceptions to this rule include: adenoma, fatty lesions, blood, proteinaceous material, melanoma metastases and contrast agents.

114
Q

41 A patient with a known colorectal carcinoma was being investigated for possible liver metastases. What modality is most sensitive in detecting liver metastases?

a Contrast-enhanced CT

b ERCP

C MRI

d FDG PET-CT

e Ultrasound

A

41 Answer C: MRI

Sensitivity for detection of liver metastases: MR > CECT > FDG PET-CT.

115
Q

42 A middle-aged man, known to have acute leukaemia, was referred for an abdominal ultrasound. Several small, uniformly hypoechoic nodules were visible within the liver, which were thought to represent foci of hepatic candidiasis and resolved following empirical treatment with antifungal drugs. Via what route is it most likely that the fungal infection spread to the liver?

a Biliary ducts

b Hepatic artery

C Percutaneous

d Portal vein

e Transcoelomic

A

42 Answer B: Hepatic artery

Unlike pyogenic and amoebic infections, which usually enter the liver via the biliary ducts or portal vein, fungal infections typically enter via the hepatic artery.

116
Q

43 A 30-year-old man with known polycythaemia rubravera was referred for contrast-enhanced CT for investigation of progressive abdominal distension and jaundice. This showed ascites, caudate and right hepatic lobar hypertrophy with mosaic enhancement of the liver. There was also a low attenuation 2-cm lesion in segment 2 which showed peripheral nodular enhancement. A delayed phase scan showed filling in of the segment 2 lesion, but the remaining liver enhancement was markedly delayed. What is the most likely diagnosis?

a Hepatocellular carcinoma

b Liver metastases

c Cirrhotic liver disease

d Budd-Chiari syndrome

e Alcoholic hepatitis

A

43 Answer D: Budd-Chiari syndrome.

117
Q

56 A three-year-old boy presents with an acute onset of generalised abdominal pain. On examination he is jaundiced with tender hepatomegaly and ascites and is diagnosed as being in acute hepatic failure. A contrast-enhanced CT scan reveals an enlarged liver with increased enhancement of the caudate lobe and patchy enhancement of the other hepatic segments. Non-enhancing thrombus is seen within the hepatic veins. Ascites and gallbladder wall oedema are present. Which of the following is the most likely diagnosis?

a Hepatocellular carcinoma

b Hepatoblastoma

C Budd-Chiari syndrome

d Hepatic veno-occlusive disease

e Wilson’s disease

A

56 Answer C: Budd-Chiari syndrome

This is the syndrome of clinical and pathological abnormalities seen with acute hepatic vein occlusion. Most cases are idiopathic but it can be related to radiation, chemotherapy, toxins and other causes. Hepatic veno-occlusive disease refers to occlusion of the post-sinusoidal venules with the inferior vena cava and major hepatic veins remaining patent.

118
Q
  1. An 18-year-old woman who is 12 weeks pregnant has a routine ultrasound examination for non-specific abdominal pain. The scan demonstrates a 1.5 cm well-defined, hyperechoic, lobulated and homogenous lesion in the right lobe of the liver. Doppler ultrasound shows no significant signal within lesion. The most likely cause is?

(a) Metastasis

(b) Haemangioma

(c) Focal nodular hyperplasia

(d) Hepatic adenoma

(e) Focal fat deposition

A
  1. (b) Haemangioma

This is the commonest lesion seen with described appearance on ultrasound. Metastasis is also less likely given the age and features. Focal fatty deposition usually has angular margins and a ‘geographic appearance’ on ultrasound. Focal nodular hyperplasia has similar reflectivity to liver on ultrasound and may show mass effect. A Doppler signal can often be seen in the lesion.

119
Q
  1. A 40-year-old woman who takes contraceptive pills is admitted with right upper quadrant pain and menorrhagia. She had a skin lesion excised by a dermatologist 5 years previously. CT shows multiple enhancing lesions in the liver. MRI demonstrates multiple homogenous hepatic lesions with increased signal on T1 images. The most likely diagnosis is?

(a) Bronchogenic carcinoma with hepatic metastases

(b) Melanoma metastases

(c) Colorectal metastases

(d) Multiple haemangiomas

(e) Hepatic adenomas

A
  1. (b) Malignant melanoma with hepatic metastases

Melanoma metastases show increased signal on T1 because of the paramagnetic effects of melanin itself. Most liver metastases show low signal on non-contrast T1 images unless there is intralesional haemorrhage, when they can be seen as heterogenous lesions.

120
Q
  1. A 60-year-old heavy goods vehicle driver presents with abdominal pain. Ultrasound shows a 5 cm area of increased echogenicity adjacent to the gallbladder. T1 in-phase gradient-echo MRIs show the lesion isointense to the liver parenchyma while the T1 out-of-phase images demonstrate a focal area of signal loss in the region. The most likely diagnosis is?

(a) Focal nodular hyperplasia

(b) Focal hepatic steatosis.

(c) Haemangioma

(d) Metastatic deposit from melanoma

(e) Focal fatty sparing

A
  1. (b) Focal hepatic steatosis

Focal fatty infiltration is typically diagnosed with in-phase and out-of-phase MRI, where the area involved demonstrates signal loss in out-of-phase imaging. This can be due to many causes including diabetes, alcoholism and obesity. Common locations include tip of segment 4, along the ligament teres and adjacent to the gallbladder. On MRI, this feature is not seen in any of the other conditions listed.

121
Q
  1. A 40-year-old white man with insulin-dependent diabetes and hyperpigmentation presents with renal colic. A non-contrast CT scan is performed which shows generalised increase in liver attenuation. Later, MRI shows homogenous signal lossin the liver on T2. The most likely cause of liver appearances is?

(a) Diffuse fatty liver

(b) Biliary cirrhosis

(c) Thorotrast injection previously

(d) Haemochromatosis

(e) Normal variant

A
  1. (d) Haemochromatosis

In haemochromatosis there is excess deposition of iron in the liver resulting in increased density of the liver on CT and loss of T2 signal due to the paramagnetic effects of iron. This also causes skin pigmentation, diabetes and arthralgia.

122
Q
  1. A 35-year-old mother on oral contraceptives presents with vague abdominal pain. Ultrasound was normal. MRI demonstrates a 3 cm, well-defined lesion in the right lobe of liver that is homogenous except for a central scar. The lesion returns low signal on T1 and slightly higher signal on T2 as compared with liver. With gadolinium, the lesion shows enhancement in arterial phase and is isointense in portovenous phase. The central scar shows late and prolonged enhancement. The most likely diagnosis is?

(a) Hepatic adenoma

(b) Hepatocellular carcinoma

(c) Giant cavernous haemangioma

(d) Focal nodular hyperplasia

(e) Metastasis

A
  1. (d) Focal nodular hyperplasia.

These are the typical features of focal nodular hyperplasia as seen on MRI. Adenoma is heterogenous on all sequences and can be indistinguishable from hepatocellular carcinoma due to haemorrhage and necrosis. Haemangioma shows peripheral nodular enhancement with centripetal filling. Metastases are commonly heterogenous

123
Q
  1. A 60-year-old diabetic man presents with fever, abdominal pain and jaundice. Ultrasound shows a 4 cm, well-defined, hypoechoic lesion with debris in the right lobe. CT shows a ill-defined, low-attenuating lesion with enhancing margins. The most likely diagnosis is?

(a) Simple liver cyst

(b) Haemangioma

(c) Pyogenic liver abscess

(d) Amoebic liver abscess

(e) Hydatid cyst

A
  1. (c) Pyogenic liver abscess

A history of fever with the described imaging features is suggestive of this diagnosis. Haemangioma is hyperechoic on ultrasound and shows centripetal filling with contrast. Amoebic abscess are uncommon in the UK and may show septations with nodular walls. On ultrasound and CT a hydatid cyst shows septations, daughter cysts, debris and calcifications. A simple liver cyst has no debris and shows no marginal enhancement.

124
Q
  1. A chronic alcoholic presents with gradually increasing abdominal distension and pedal oedema. Ultrasound shows a small nodular liver and ascites, and Doppler shows absence of blood flow in the left and middle hepatic veins. What is the most likely cause of absence of blood flow in hepatic veins?

(a) Budd–Chiari syndrome

(b) Cirrhosis of the liver

(c) Hepatoma

(d) Cavernous transformation of the portal vein

(e) None of the above

A
  1. (a) Budd–Chiari syndrome

This is caused by obstruction of one or more hepatic veins or the inferior vena cava. If the hepatic veins are seen on the grey scale but no flow is identified and reversed flow is not seen, Budd–Chiari syndrome is likely. There are numerous causes for Budd–Chiari syndrome, but most commonly it is idiopathic.

125
Q
  1. Regarding hepatocellular carcinoma: (T/F)

(a) Haemochromatosis is a recognised cause.

(b) It is the commonest primary visceral malignancy in the world.

(c) Elevated alpha-fetoprotein is found in 50-60% of cases.

(d) Has a higher incidence in macronodular than micronodular cirrhosis.

(e) On MR, hepatoma has a well defined, hypointense capsule on T1 weighted images.

A

Answers:

(a) Correct
(b) Correct
(c) Not correct
(d) Correct
(e) Not correct

Explanation:

Causes of hepato-cellular carcinoma are haemochromatosis, cirrhosis, hepatitis, Wilson’s disease, alpha 1 antitrypsin deficiency. Elevated alpha feto-protein levels are found in 50-60% cases of hepato-cellular carcinoma. On MRI, hepatoma shows increased signal intensity on T2-weighted images with peripheral gadolinium enhancement in 20% of the cases.

126
Q
  1. Regarding Budd-Chiari syndrome: (T/F)

(a) It can be caused by obstruction of the suprahepatic IVC.

(b) On early CT images, the central liver enhances
prominently and the peripheral liver weakly.

(c) The caudate lobe is markedly atrophic.

(d) A ‘spider’s web’ appearance at hepatic venography characteristic.

(e) On MRI images ‘comma-shaped’ intrahepatic collateral vessels are seen.

A

Answers:

(a) Correct
(b) Correct
(c) Not correct
(d) Correct
(e) Correct

Explanation:

Caudate lobe is enlarged and hypertrophied in Budd chiari syndrome. Flip-flop enhancement pattern is seen with central hepatic enhancement in the early phase and peripheral enhancement in the late phase. Thrombosis in hepatic veins is more common cause than obstruction of the suprahepatic IVC.

127
Q
  1. Regarding hepatic adenoma: (T/F)

(a) It is associated with Type 1 glycogen storage disease.

(b) Is located in the left lobe of the liver in 60-75% of cases.

(c) Is easily differentiated from hepatocellular carcinoma on MRI.

(d) Often reduces in size during pregnancy.

(e) Is hypovascular.

A

Answers:

(a) Correct
(b) Not correct
(c) Not correct
(d) Not correct
(e) Not correct

Explanation:

Hepatic adenoma increases in size during pregnancy. There are hypervascular lesions with significant risk of bleeding at the biopsy and hence biopsy is contraindicated. There are indistinguishable on all MRI sequences. There mostly located in the right lobe of liver in 60-75% of the cases.

128
Q
  1. Focal nodular hyperplasia (FNH): (T/F)

(a) Multiple lesions are seen in 40-60% of cases.

(b) Is the most common benign liver tumour.

(c) Has a low signal on T2 weighted MRI post-iron oxide administration.

(d) Central scar is a pathognomonic feature.

(e) Central scar is hyperdense on arterial phase CT.

A

Answers:

(a) Not correct
(b) Not correct
(c) Correct
(d) Not correct
(e) Not correct

Explanation:

FNH is the second most common benign liver tumour after haemangioma. The usually occurs in young patients with more female ratio. There are multiple in 20% of the cases. A central scar is usually seen in FNH however it can be seen in giant haemangioma, hepato cellular carcinoma and hepatic adenoma. Central scar of FNH is non-enhancing on arterial phase however the rest of the tumour enhances significantly.

129
Q
  1. Complications of liver transplantation: (T/F)

(a) Hepatic artery thrombosis occurs in less than 1% of transplant recipients

(b) Portal vein thrombosis is the most common vascular complication.

(c) Non-anastamotic biliary strictures carry a worse prognosis than anastamotic strictures.

(d) Post-transplant lymphoproliferative disorder is frequently associated with Epstein-Barr virus infection.

(e) Abscess formation is seen in 5-10%.

A

Answers:

(a) Not correct
(b) Not correct
(c) Correct
(d) Correct
(e) Correct

Explanation:

Most common vascular complication of liver transplant includes hepatic artery thrombosis which occurs in 60% of the cases of vascular complications. It occurs within 15 days of transplantation. Portal vein thrombosis occurs in 1-2% cases of liver transplant.

130
Q
  1. Which of the following are true regarding fibrolamellar hepatocellular carcinoma? (T/F)

(a) The majority are associated with elevated alpha fetoprotein.

(b) A central scar is present in 50% of cases on ultrasonography.

(c) There is delayed enhancement of the tumour following intravenous gadolinium on MRI.

(d) Calcification is rarely seen on CT.

(e) The central scar may enhance following intravenous contrast on CT.

A

Answers:

(a) Not correct
(b) Correct
(c) Not correct
(d) Not correct
(e) Correct

Explanation:

Fibrolamellar carcinoma shows calcification in 40% of the cases. The tumour is unusually large, hypodense on unenhanced CT and hyperdense during the arterial phase following contrast administration. The not associated with elevated alpha-fetoprotein levels and chronic liver disease. On MRI, these tumours are heterogeneously hypointense on T1 and hyperintense on T2-weighted images.

131
Q
  1. Which of the following are correct regarding imaging of liver transplantation? (T/F)

(a) Hepatic artery thrombosis is the most common vascular complication.

(b) Portal vein thrombosis usually occurs within 24 hours of transplantation.

(c) Hepatic vein stenosis is more common following living related transplants than after cadaveric transplants.

(d) Biliary anastamotic stenosis are reliably diagnosed using magnetic resonance cholangiopancreatography (MRCP).

(e) Periportal low attenuation on contrast-enhanced CT is a reliable sign of acute graft rejection.

A

Answers:

(a) Correct
(b) Not correct
(c) Correct
(d) Not correct
(e) Not correct

Explanation:

Periportal low attenuation on contrast enhanced CT may be a feature of acute graft rejection, but it has poor sensitivity and specificity and is frequently seen with oedema of the periportal lymphatic vessels. Portal vein stenosis or thrombosis developsslowly, presenting with varices, splenomegaly and ascites. Portal vein stenosis may be treated by balloon dilatation, but once the thrombus is extensive and reaches the periphery of the intrahepatic portal vein branches, then a repeat liver transplant is only option.